Вы находитесь на странице: 1из 69

Representation Theory

Lectured by S. Martin
Lent Term 2009, 2010, 2011
1 Group Actions 1
2 Linear Representations 3
3 Complete Reducibility and Maschkes Theorem 7
4 Schurs Lemma 10
5 Character Theory 13
6 Proofs and Orthogonality 17
7 Permutation Representations 20
8 Normal Subgroups and Lifting Characters 23
9 Dual Spaces and Tensor Products of Representations 25
10 Induction and Restriction 32
11 Frobenius Groups 37
12 Mackey Theory 39
13 Integrality 41
14 Burnsides p
a
q
b
Theorem 44
15 Representations of Topological Groups 46
Examples Sheets
REPRESENTATION THEORY (D) 24 lectures, Lent term
Linear Algebra, and Groups, Rings and Modules are essential.
Representations of nite groups
Representations of groups on vector spaces, matrix representations. Equivalence of represen-
tations. Invariant subspaces and submodules. Irreducibility and Schurs Lemma. Complete
reducibility for nite groups. Irreducible representations of Abelian groups.
Characters
Determination of a representation by its character. The group algebra, conjugacy classes, and
or- thogonality relations. Regular representation. Induced representations and the Frobenius
reciprocity theorem. Mackeys theorem. [12]
Arithmetic properties of characters
Divisibility of the order of the group by the degrees of its irreducible characters. Burnsides
p
a
q
b
theorem. [2]
Tensor products
Tensor products of representations. The character ring. Tensor, symmetric and exterior
algebras. [3]
Representations of S
1
and SU
2
The groups S
1
and SU
2
, their irreducible representations, complete reducibility. The
Clebsch-Gordan formula. *Compact groups.* [4]
Further worked examples
The characters of one of GL
2
(F
q
), S
n
or the Heisenberg group. [3]
Appropriate books
J.L. Alperin and R.B. Bell Groups and representations. Springer 1995 (37.50 paperback).
I.M. Isaacs Character theory of nite groups. Dover Publications 1994 (12.95 paperback).
G.D. James and M.W. Liebeck Representations and characters of groups. Second Edition,
CUP 2001 (24.99 paperback).
J-P. Serre Linear representations of nite groups. Springer-Verlag 1977 (42.50 hardback).
M. Artin Algebra. Prentice Hall 1991 (56.99 hardback).
Representation Theory
This is the theory of how groups act as groups of transformations on vector spaces.
group (usually) means nite group.
vector spaces are nite-dimensional and (usually) over C.
1. Group Actions
F a eld usually F = C or R or Q : ordinary representation theory;
sometimes F = F
p
or F
p
(algebraic closure) : modular representation theory.
V a vector space over F always nite-dimensional over F.
GL(V ) = : V V , linear, invertible group operation is composition, identity is 1.
Basic linear algebra
If dim
F
V = n < , choose a basis e
1
, . . ., e
n
over F so that we can identify it with F
n
.
Then GL(V ) corresponds to a matrix A

= (a
ij
) F
nn
where (e
j
) =

i
a
ij
e
i
, and
A

GL
n
(F), the general linear group.
(1.1) GL(V )

= GL
n
(F), A

. (A group isomorphism check A


12
= A
1
A
2
, bijection.)
Choosing dierent bases gives dierent isomorphisms to GL
n
(F), but:
(1.2) Matrices A
1
, A
2
represent the same element of GL(V ) with respect to dierent bases
i they are conjugate/similar, viz. there exists X GL
n
(F) such that A
2
= XA
1
X
1
.
Recall the trace of A, tr (A) =

i
a
ii
where A = (a
ij
) F
nn
.
(1.3) tr (XAX
1
) = tr (A), hence dene tr () = tr (A), independent of basis.
(1.4) Let GL(V ) where V is nite-dimensional over C and is idempotent, i.e.

m
= id, some m. Then is diagonalisable. (Proof uses Jordan blocks see Telemann p.4.)
Recall End(V ), the endomorphism algebra, is the set of all linear maps V V with
natural addition of linear maps, and the composition as multiplication.
(1.5) Proposition. Take V nite-dimensional over C, End(V ). Then is diagonalis-
able i there exists a polynomial f with distinct linear factors such that f() = 0.
Recall in (1.4),
m
= id, so take f = X
m
1 =

m1
j=0
(X
j
), where = e
2i/n
.
Proof of (1.5). f(X) = (X
1
). . .(X
k
).
Let f
j
(X) =
(X
1
). . .

(X
j
). . .(X
k
)
(
j

1
). . .

(
j

j
). . .(
j

k
)
, where means remove.
So 1 =

f
j
(X). Put V
j
= f
j
()V . The f
j
() are orthogonal projections, and V =

V
j
with V
j
V (
j
) the
k
-eigenspace.
1
(1.4*) In fact, a nite family of commuting separately diagonalisable automorphisms of a
C-space can be simultaneously diagonalised.
Basic group theory
(1.6) Symmetric group, S
n
= Sym(X
n
) on the set X
n
= 1, . . ., n, is the set of all
permutations (bijections X
n
X
n
) of X
n
. [S
n
[ = n!
Alternating group, A
n
on X
n
, is the set of products of an even number of transpositions
(ij) S
n
. (A
n
is mysterious. Results true for S
n
usually fail for A
n
!)
(1.7) Cyclic group of order n, C
n
= x : x
n
= 1). E.g., Z/nZ under +.
Its also the group of rotations, centre 0, of the regular n-gon in R
2
. And also the group of
n
th
roots of unity in C (living in GL
1
(C)).
(1.8) Dihedral group, D
2m
of order 2m. D
2m
= x, y : x
m
= y
2
= 1, yxy
1
= x
1
).
Can think of this as the set of rotations and reections preserving a regular m-gon (living in
GL
2
(R)). E.g., D
8
, of the square.
(1.9) Quaternion group, Q
8
= x, y : x
4
= 1, y
2
= x
2
, yxy
1
= x
1
) of order 8.
(Often used as a counterexample to dihedral results.)
In GL
2
(C), can put x =
_
i 0
0 i
_
, y =
_
0 1
1 0
_
.
(1.10) The conjugacy class of g G is (
G
(g) = xgx
1
: x G.
Then [(
G
(g)[ = [G : C
G
(g)[, where C
G
(g) = x G : xg = gx is the centraliser of g G.
Denition. G a group, X a set. G acts on X if there exists a map : G X X,
(g, x) g x, written gx for g G, x X, such that:
1x = x for all x X
(gh)x = g(hx) for all g, h G, x X.
Given an action of G on X, we obtain a homomorphism : G Sym(X) called the per-
mutation representation of G.
Proof. For g G, the function
g
: X X, x gx, is a permutation (inverse is
g
1 ).
Moreover, for all g
1
, g
2
G,
g1g2
=
g1

g2
since (g
1
g
2
)x = g
1
(g
2
x) for x X.
In this course, X is often a nite-dimensional vector space, and the action is linear, viz:
g(v
1
+v
2
) = gv
1
+gv
2
, g(v) = gv for all v, v
1
, v
2
V = X, g G, F.
2
2. Linear Representations
G a nite group. F a eld, usually C.
(2.1) Denition. Let V be a nite-dimensional vector space over F. A (linear) repre-
sentation of G on V is a homomorphism =
V
: G GL(V ).
We write
g
for
V
(g). So for each g G,
g
GL(V ) and
g1g2
=
g1

g2
.
The dimension or degree of is dim
F
V .
(2.2) Recall ker G and G/ ker

= (G) GL(V ). (The rst isomorphism theorem.)
We say that is faithful if ker = 1.
Alternative (and equivalent) approach:
(2.3) G acts linearly on V if there exists a linear action GV V , viz:
action: (g
1
g
2
)v = g
1
(g
2
v), 1v = v, for all g
1
, g
2
G, v V
linearity: g(v
1
+v
2
) = gv
1
+gv
2
, g(v) = gv, for all g G, v V , F.
So if G acts linearly on V , the map G GL(V ), g
g
, with
g
: v gv, is a represen-
tation of V . And conversely, given a representation G GL(V ), we have a linear action of
G on V via g.v = (g)v, for all v V , g G.
(2.4) In (2.3) we also say that V is a G-space or a G-module. In fact, if we dene the
group algebra FG =
_
gG

g
g :
g
F
_
then V is actually an FG-module.
Closely related:
(2.5) R is a matrix representation of G of degree n if R is a homomorphism G GL
n
(F).
Given a linear representation : G GL(V ) with dim
F
V = n, x basis B; get a matrix
representation G GL
n
(F), g [(g)]
B
.
Conversely, given matrix representation R : G GL
n
(F), we get a linear representation
: G GL(V ), g
g
, via
g
(v) = R
g
(v).
(2.6) Example. Given any group G, take V = F (the 1-dimensional space) and : G
GL(V ), g (id : F F). This is known as the trivial/principal representation. So
deg = 1.
(2.7) Example. G = C
4
= x : x
4
= 1).
Let n = 2 and F = C. Then R : x X (some matrix X) will determine all x
j
X
j
. We
need X
4
= I. We can take X diagonal with diagonal entries 1, i (16 choices). Or we
can take X not diagonal, then it will be isomorphic to some diagonal matrix, by (1.4).
(2.8) Denition. Fix G, F. Let V, V

be F-spaces and : G
GL(V ),

: G GL(V

) be representations of G. The linear map


: V V

is a G-homomorphism if
() (g) =

(g) for all g G.


V

V

V

g
V

the square commutes


3
We say intertwines ,

.
We write Hom
G
(V, V

) for the F-space of all of these.


We say that is a G-isomorphismif also is bijective; if such a exists we say that ,

are
isomorphic. If is a G-isomorphism, we write () as

=
1
(meaning

(g) = (g)
1
for all g G).
(2.9) The relation of being isomorphic is an equivalence relation on the set of all linear
representations of G (over F).
Remark. The basic problem of representation theory is to classify all representations of
a given group G up to isomorphisms. Good theory exists for nite groups over C, and for
compact topological groups.
(2.10) If ,

are isomorphic representations, they have the same dimension. Converse is


false: in C
4
there are four non-isomorphic 1-dimensional representations. If = e
2i/4
then
we have
j
(
i
) =
ij
(0 i 3).
(2.11) Given G, V over F of dimension n and : G GL(V ). Fix
a basis B for V ; we get a linear isomorphism : V F
n
, v [v]
B
.
Get a representation

: G GL(F
n
) isomorphic to .
V

V

F
n

F
n
(2.12) In terms of matrix representations, R : G GL
n
(F), R

: G GL
n
(F) are G-
isomorphic if there exists a (non-singular) matrix X GL
n
(F) with R

(g) = XR(g)X
1
(for all g G).
In terms of G-actions, the actions of G on V , V

are G-isomorphic if there is an isomorphism


: V V

such that g (v)


..
in V

= (gv)
..
in V
for all g G, v V .
Subrepresentations
(2.13) Let : G GL(V ) be a representation of G. Say that W V is a G-subspace if
its a subspace and is (G)-invariant, i.e.
g
(W) W for all g G. E.g., 0 and V .
Say is irreducible, or simple, if there is no proper G-subspace.
(2.14) Example. Any 1-dimensional representation of G is irreducible. (But not conversely:
e.g. D
6
has a 2-dimensional C-irreducible representation.)
(2.15) In denition (2.13) if W is a G-subspace then the corresponding map G GL(W),
g (g)[
W
is a representation of G, a subrepresentation of .
(2.16) Lemma. : G GL(V ) a representation. If W is a G-subspace of V and if
B = v
1
, . . ., v
n
is a basis of V containing the basis v
1
, . . ., v
m
of W, then the matrix of
(g) with respect to B is (with the top-left
*
being mm)
_
* *
0
*
_
(for each g G)
4
(2.17) Examples
(i) (2.10) revisited. The irreducible representations of C
4
= x : x
4
= 1) are all 1-
dimensional, and four of these are x i, x 1, x i, x 1.
(The two x i are faithful, since they have trivial kernel.)
In general, C
m
= x : x
m
= 1) has precisely m irreducible complex representations, all
of degree 1. Put = e
2i/m

m
and dene
k
by
k
: x
j

jk
(0 j, k m1).
It turns out that all irreducible complex representations of a nite abelian group are
1-dimensional: (1.4*) or see (4.4) below.
(ii) G = D
6
= x, y : x
3
= y
2
= 1, yxy
1
= x
1
), the smallest non-abelian nite group.
G

= S
3
(generated by a 3-cycle and a 2-cycle).
G has the following irreducible complex representations:
2 of degree 1 :
1
: x 1, y 1

2
: x 1, y 1
1 of degree 2 :
3
: x
_
0
0
1
_
, y
_
0 1
1 0
_
, where = e
2i/3

3
This follows easily later on. For now, by brute force. . .
Dene u
0
= 1 +x +x
2
, v
0
= u
0
y,
u
1
= 1 +
2
x +x
2
, v
1
= u
1
y,
u
2
= 1 +x +
2
x
2
, v
2
= u
2
y.
Check easily xu
1
= x+
2
x
2
+ = u
1
, and in general xu
i
=
i
u
i
(0 i 2). (I.e., in
the action of x, u
i
is an eigenvector, of eigenvalue
i
.) So u
i
), v
i
) are Cx)-modules.
Also: yu
0
= v
0
, yv
0
= u
0
,
yu
1
= v
2
, yv
1
= u
2
,
yu
2
= v
1
, yv
2
= u
1
.
_
_
_
So u
0
, v
0
), u
1
, v
2
), u
2
, v
1
) are Cy)-modules,
and hence are all CG-submodules.
Note, U
3
= u
1
, v
2
), U
4
= u
2
, v
1
) are irreducible and u
0
, v
0
) has U
1
= u
0
+ v
0
) and
U
2
= u
0
v
0
) as CG-submodules.
Moreover, CD
6
= U
1
U
2
U
3
U
4
. .
.

trivial non-trivial isomorphic via u1 v1, v2 u2
(iii) G = D
8
= x, y : x
4
= y
2
= 1, yxy
1
= x
1
), the rotations/reections of a square.
G S
4
. See (1.8) with m = 4.
G has the following irreducible complex representations:
4 of degree 1 :
0
: x 1, y 1 (trivial)

1
: x 1, y 1

2
: x 1, y 1

3
: x 1, y 1
1 of degree 2 :
3
: x
_
i 0
0 i
_
, y
_
0 1
1 0
_
(up to isomorphism)
5
By considering the eect of y on eigenvectors of x, well show that any irreducible
representation of G is isomorphic to one of the
i
. This is easy to do later. Here, let V
be some irreducible G-space.
Under the action of x, we have
V [
x
= V
1
V
1
V
i
V
i
where V

= v V : xv = v.
For the y-action: if xv = v then yv V
1
, since x(yv) = yx
1
v = yv; similarly if
xv = v then yv V
1
, since x(yv) = yx
1
v = yv.
So, if V is irreducible and V
1
,= 0 or V
1
,= 0, then V is 1-dimensional (so one of the

j
, 0 j 3).
Taking v V
1
, we have xv = v, and yv is either v or v.
Taking v V
1
, we have xv = v, and yv is either v or v.
_
So, four cases.
Final case: V = V
i
V
i
.
Let v V
i
, i.e. xv = iv. Then yv V
i
, since x(yv) = yx
1
v = iyv, and vice versa.
Clearly v, yv) is G-invariant, so V = v, yv) as V is irreducible. Taking basis v, yv
we have x, y acting as in
4
(with respect to this basis).
See James & Liebeck, p. 94, and also Example Sheet 1.
(2.18) Denition. We say that : G GL(V ) is decomposable if there are G-invariant
subspaces U, W with V = U W. Say is a direct sum
U

W
. If no such exists, we say
is indecomposable.
(U, W must have G-actions on them, not just ordinary vector subspaces.)
(2.19) Lemma. Suppose : G GL(V ) is a decomposition with G-invariant decomposition
V = U W. If B is a basis u
1
, . . ., u
k
, w
1
, . . ., w
l
consisting of a basis B
1
of U and B
2
of
W, then with respect to B,
(g)
B
=
_
*
0
0
*
_
=
_
[
U
(g)]
B1
0
0 [
W
(g)]
B2
_
(2.20) Denition. : G GL(V ),

: G GL(V

). The direct sum of ,

is

: G GL(V V

), (

)(g)(v
1
+v
2
) = (g)v
1
+

(g)v
2
a block diagonal action.
For matrix representations, R : G GL
n
(F), R

: G GL
n
(F), dene
R R

: G GL
n+n
(F), g
_
R(g) 0
0 R

(g)
_
, g G.
6
3. Complete Reducibility and Maschkes Theorem
G, F as usual.
(3.1) Denition. The representation : G GL(V ) is completely reducible, or
semisimple, if it is a direct sum of irreducible representations.
Evidently, simple completely reducible, but not conversely.
(3.2) Examples. Not all representations are completely reducible.
(i) G =
__
1 n
0 1
_
: n Z
_
, V = C
2
, natural action (gv is matrix multiplication).
V is not completely reducible. (Note G not nite.)
(ii) G = C
p
, F = F
p
. x
j

_
1 0
j 1
_
(0 j p 1) denes a representation G GL
2
(F).
V = (v
1
, v
2
) where x
j
v
1
= v
1
, x
j
v
2
= jv
1
+v
2
. Dene W = (v
1
).
Then W is an FC
p
-module but there is no X s.t. V = W X. (Note F ,= R, C.)
(3.3) Theorem (Complete Reducibility Theorem). Every nite-dimensional represen-
tation of a nite group over a eld of characteristic 0 is completely reducible.
Enough to prove the following.
(3.4) Theorem (Maschkes Theorem). G nite, : G GL(V ) with V an F-space,
char F = 0. If W is a G-subspace of V then there exists a G-subspace U of V such
that V = W U (a direct sum of G-subspaces).
Note. The proof below also works for (char F, [G[) = 1.
Proof 1. Let W

be any vector space complement of W in V , i.e. V = W W

. Let
q : V W be the projection of V onto W along W

, i.e. if v = w +w

then q(v) = w.
Dene q : v
1
[G[

gG
(g)q((g
1
)v), the average of q over G.
Drop the s i.e. write (g)q((g
1
)v) as gq(g
1
v).
Claim (i). q : V W.
For v V , q(g
1
v) W and gW W (as W is g-invariant).
Claim (ii). q(w) = w for w W.
q(w) =
1
[G[

gG
gq(g
1
w
. .
W
) =
1
[G[

gG
g(g
1
w) =
1
[G[

gG
w = w.
So (i), (ii) q projects V onto W.
7
Claim (iii). If h G then hq(v) = q(hv) (for v V ).
hq(v) = h
1
[G[

gG
gq(g
1
v) =
1
[G[

gG
hgq(g
1
v) =
1
[G[

gG
(hg)q
_
(hg)
1
hv
_
=
1
[G[

G
g

q
_
g
1
(hv)
_
= q(hv).
Claim (iv). ker q is G-invariant.
If v ker q, h G, then hq(v) = 0 = q(hv), so hv ker q.
Then V = imq ker q = W ker q is a G-subspace decomposition.
Remark. Complements are not necessarily unique.
The second proof uses inner products, hence we need to take F = C (or R), and it can be
generalised to compact groups (chapter 15).
Recall for V a C-space, , ) is a C-inner product if
(a) w, v) = v, w) for all v, w
(b) linear in LHS
(c) v, v) > 0 if v ,= 0
Additionally, , ) is G-invariant if
(d) gv, gw) = v, w) for all v, w V, g G
Note that if W is a G-subspace of V (with G-invariant inner product) then W

is also
G-invariant and V = W W

.
Proof. Want: for all v W

, for all g G, we have gv W

.
Now, v W

v, w) = 0 for all w W. Thus gv, gw) = 0 for all g G, w W.


Hence gv, w

) = 0 for all w

W since we can take w = g


1
w

by G-invariance of W.
Hence gv W

since g was arbitrary.


Hence if there is a G-invariant inner product on any complex G-space, we get:
(3.4

) (Weyls Unitary Trick). Let be a complex representation of the nite group G on


the C-space V . There is a G-invariant inner product on V (whence (G) is conjugate
to a subgroup of U(V ), the unitary group on V , i.e. (g)

= (g
1
)).
Proof. There is an inner product on V : take basis e
1
, . . ., e
n
, and dene (e
i
, e
j
) =
ij
,
extended sesquilinearly. Now dene v, w) =
1
[G[

gG
(gv, gw).
Claim. , ) is sesquilinear, positive denite, and G-invariant.
If h G, hv, hw) =
1
[G[

gG
_
(gh)v, (gh)w
_
=
1
[G[

G
(g

v, g

w) = v, w).
8
(3.5) (The (left) regular representation of G.) Dene the group algebra of G to be the
F-space FG = spane
g
: g G.
There is a G-linear action: for h G, h

g
a
g
e
g
=

g
a
g
e
hg
_
=

g
a
h
1
g
e
g

_
.

reg
is the corresponding representation the regular representation of G.
This is faithful of dimension [G[.
It turns out that every irreducible representation of G is a subrepresentation of
reg
.
(3.6) Proposition. Let be an irreducible representation of the nite group G over a eld
of characteristic 0. Then is isomorphic to a subrepresentation of
reg
.
Proof. Take : G GL(V ), irreducible, and let 0 ,= v V .
Let : FG V ,

g
a
g
e
g

a
g
gv (a G-homomorphism).

really (g)
Now, V is irreducible and im = V (since im is a G-subspace). Then ker is a
G-subspace of FG. Let W be a G-complement of ker in FG (using (3.4)), so that
W < FG is a G-subspace and FG = ker W.
Hence W

= FG/ ker

= im = V .

G-isom.
More generally,
(3.7) Denition. Let F be a eld, and let G act on a set X. Let FX = spane
x
: x X,
with G-action g
_
xX
a
x
e
x
_
=

a
x
e
gx
.
So we have a G-space FX. The representation G GL(V ) with V = FX is the correspond-
ing permutation representation.
9
4. Schurs Lemma
(4.1) Theorem (Schurs Lemma). (a) Assume V, W are irreducible G-spaces (over a
eld F). Then any G-homomorphism : V W is either 0 or is an isomorphism.
(b) Assume F is algebraically closed and let V be an irreducible G-space. Then any
G-endomorphism : V V is a scalar multiple of the identity map id
V
(a homothety).
Proof. (a) Let : V W be a G-homomorphism. Then ker is a G-subspace of V , and
since V is irreducible either ker = 0 or ker = V . And im is a G-subspace of W,
so as W is irreducible, im is either 0 or W. Hence either = 0 or is injective and
surjective, so is an isomorphism.
(b) Since F is algebraically closed, has an eigenvalue . Then id is a singular
G-endomorphism on V , so must be 0, so = id.
Recall from (2.8) the F-space Hom
G
(V, W) of all G-homomorphisms V W. Write
End
G
(V ) for the endomorphism algebra Hom
G
(V, V ).
(4.2) Corollary. If V, W are irreducible complex G-spaces, then
dim
C
Hom
G
(V, W) =
_
1 if V, W are G-isomorphic
0 otherwise
Proof. If V, W are not isomorphic then the only G-homomorphism V W is 0 by (4.1).
Assume V

=G
W and
1
,
2
Hom
G
(V, W), both ,= 0. Then
2
is irreducible by (4.1)
and
1
2

1
Hom
G
(V, V ). So
1
2

1
= id for some C. Then
1
=
2
.
(4.3) Corollary. If G has a faithful complex irreducible representation then Z(G) is cyclic.
Remark. The converse is false. (See examples sheet 1, question 11.)
Proof. Let : G GL(V ) be a faithful irreducible complex representation. Let z Z(G),
so zg = gz for all g G.
Consider the map
z
: v zv for v V . This is a G-endomorphism on V , hence is
multiplication by a scalar
z
, say (by Schur).
Then the map Z(G) C

, z
z
, is a representation of Z and is faithful (since
is). Thus Z(G) is isomorphic to a nite subgroup of C

, hence is cyclic.
Applications to abelian groups
(4.4) Corollary. The irreducible complex representations of a nite abelian group G are all
1-dimensional.
Proof. Either (1.4*) to invoke simultaneous diagonalisation: if v is an eigenvector for each
g G and if V is irreducible, then V = (v).
Or let V be an irreducible complex representation. For g G, the map
g
: V V

,
v gv, is a G-endomorphism of V and, as V is irreducible,
g
=
g
id for some
g
C.
Thus gv =
g
v for any g. Thus, as V is irreducible, V = (v) is 1-dimensional.
Remark. This fails on R. E.g., C
3
has two irreducible real representations: one of dimension
1, one of dimension 2. (See sheet 1, question 12.)
10
Recall that any nite abelian group G is isomorphic to a product of cyclic groups, e.g.
C
6

= C
2
C
3
. In fact, it can be written as a product of C
p
for various primes p and 1,
and the factors are uniquely determined up to ordering.
(4.5) Proposition. The nite abelian group G = C
n1
. . .C
nr
has precisely [G[ irreducible
complex representations, as described below.
Proof. Write G = x
1
) . . . x
r
) where o(x
j
) = n
j
. Suppose is irreducible so by (4.4)
its 1-dimensional, : G C

.
Let (1, . . ., 1, x
j
, 1, . . ., 1) =
j
C

. Then
nj
j
= 1, so
j
is an n
th
j
root of unity.
Now the values (
1
, . . .,
r
) determine , as (x
j1
1
, . . ., x
jr
r
) =
j1
1
. . .
jr
r
.
Thus (
1
, . . .,
r
) with
nj
j
= 1 for all j. (And have n
1
. . .n
r
such r-tuples, each
giving a 1-dimensional representation.
Examples. (a) G = C
4
= x). 1 x x
2
x
3

1
1 1 1 1

2
1 i 1 i

3
1 1 1 1

4
1 i 1 i
(b) G = V
4
= x
1
) x
2
)

= C
2
C
2
. 1 x
1
x
2
x
1
x
2

1
1 1 1 1

2
1 1 1 1

3
1 1 1 1

4
1 1 1 1
Warning. There is no natural 1-1 correspondence between the elements of G and the
representations of G. If you choose an isomorphism G

= C
1
. . . C
r
, then you can identify
the two sets, but it depends on the choice of isomorphism.
** Non-examinable section **
Application to isotypical decompositions
(4.6) Proposition. Let V be a G-space over C, and assume V = U
1
. . . U
n
= W
1
. . .
W
n
, with all the U
j
, W
k
irreducible G-spaces. Let X be a xed irreducible G-space.
Let U be the sum of all the U
j
isomorphic to X, and W be the sum of all the W
j
isomorphic to X.
Then U = W, and is known as the isotypical component of V corresponding to X.
Hence:
# U
j
isomorphic to X = # W
k
isomorphic to X = (V : X) = multiplicity of X in V .
Proof (sketch). Look at
jk
: U
j
ij
V

k
W
k
, with W
k

= X, where i
j
is inclusion and
k
is projection. If U
j

= X then U
j
W all the projections to the other W
l
are 0.
Then ddle around with dimensions, then done.
11
(4.7) Proposition. (V : X) = dim
C
Hom
G
(V, X) for X irreducible, V any G-space.
Proof. Prove Hom
G
(W
1
W
2
, X)

= Hom
G
(W
1
, X) Hom
G
(W
2
, X), then apply Schur.
(Or see James & Liebeck 11.6.)
(4.8) Proposition. dim
C
Hom
G
(CG, X) = dim
C
X.
Proof. Let d = dimX, and take a basis e
1
, . . ., e
d
of X. Dene
i
: CG X, g ge
i
(1 i d). Then
i
Hom
G
(CG, X) and
1
, . . .,
d
is a basis. (See James &
Liebeck 11.8.)
Remark. If V
1
, . . ., V
r
are all the distinct complex irreducible G-spaces then CG = n
1
V
1

. . .n
r
V
r
where n
i
= dimV
i
. Then [G[ = n
2
1
+. . .+n
2
r
. (See (5.9), or James & Liebeck 11.2.)
Recall (2.17). G = D
6
, CG = U
1
U
3
U
3
U
4
, dimHom(CG, U
3
) = 2. (Challenge: nd a
basis for it.) U
1
and U
2
occur with multiplicity 1, and U
3
occurs with multiplicity 2.
** End of non-examinable section **
12
5. Character Theory
We want to attach invariants to a representation of a nite group Gon V . Matrix coecients
of (g) are basis dependent, so not true invariants.
Take F = C, and G nite. =
V
: G GL(V ), a representation of G.
(5.1) Denition. The character

=

V
=

is dened by

(g) = tr (g) (= tr R(g),
where R(g) is any matrix representation of (g) with respect to any basis). The degree of

V
is dimV .
Thus

is a function G C.

is linear if dimV = 1, in which case

is a homomorphism
G C


is irreducible if is.


is faithful if is.


is trivial (principal) if is the trivial representation: write

= 1
G
.

is a complete invariant in the sense that it determines up to isomorphism see (5.7).


(5.2) First properties.
(i)

V
(1) = dimV .
(ii)

V
is a class function, viz it is conjugation invariant, i.e.

V
(hgh
1
) =

V
(g)
for all g, h G.
Thus

V
is constant on the conjugacy classes (ccls) of G.
(iii)

V
(g
1
) =

V
(g).
(iv) For two representations, V, W, have

V W
=

V
+

W
.
Proof. (i) tr (I) = n.
(ii)

(hgh
1
) = tr (R
h
R
g
R
h
1 ) = tr (R
g
) =

(g).
(iii) g G has nite order, so by (1.4) can assume (g) is represented by a diagonal ma-
trix
_

1
.
.
.

n
_
. Thus

(g) =

i
. Now g
1
is represented by
_

1
1
.
.
.

1
n
_
.
and since [
i
[ = 1 for all i,

(g
1
) =

1
i
=

i
=

i
=

(g).
(iv) Suppose V = V
1
V
2
,
i
: G GL(V
i
), : G GL(V ). Take basis B = B
1
B
2
of V , containing bases B
i
of V
i
.
With respect to B, (g) has matrix
_
[
1
(g)]
B1
0
0 [
2
(g)]
B2
_
.
So

(g) = tr (this) = tr
1
(g) + tr
2
(g) =

1
(g) +

2
(g).
Remark. We see later that

1
,

2
characters of G

1

2
also a character of G. This uses
tensor products see (9.6).
(5.3) Lemma. Let : G GL(V ) be a complex representation aording (which can
take) the character

. Then [

(g)[

(1), with equality i (g) = id for some
C, a root of unity. Moreover,

(g) =

(1) g ker .
Proof. Fix g. W.r.t. a basis of V of eigenvectors of (g), the matrix of (g) is
_

1
.
.
.

n
_
.
13
Hence, [

(g)[ = [

i
[

[
i
[ =

1 = dimV =

(1), with equality i all
j
are
equal to , say (using Cauchy-Schwarz). And if

(g) =

(1) then (g) = id.
Therefore,

(g) =

(1), and so = 1 and g ker .


(5.4) Lemma. If

is a complex irreducible character of G, then so is

, and so is

for
any linear character of G.
Proof. If R : G GL
n
(C) is a complex (matrix) representation then so is R : G GL
n
(C),
g R(g).
Similarly for R

: g (g)R(g). Check the details.


(5.5) Denition. ((G) = f : G C : f(hgh
1
) = f(g) h, g G, the C-space of class
functions. (Where f
1
+f
2
: g f
1
(g) +f
2
(g), f : g f(g).)
List conjugacy classes as (
1
(= 1), (
2
, . . ., (
k
. Choose g
1
(= 1), g
2
, . . ., g
k
as representatives
of the classes.
Note also that dim
C
((G) = k, as the characteristic functions
j
of the conjugacy classes form
a basis, where
j
(g) = 1 if g (
j
, and 0 otherwise.
Dene Hermitian inner product on ((G) by
f, f

) =
1
[G[

gG
f(g)f

(g) =
1
[G[
k

j=1
[(
j
[ f(g
j
)f

(g
j
) =
k

j=1
1
[C
G
(g
j
)[
f(g
j
)f

(g
j
)
using orbit-stabilier: [([ = [G : C
G
(x)[, where C
G
(x) is the centraliser of x in G.
For characters,

) =
k

j=1
1
[C
G
(g
j
)[

(g
1
j
)

(g
j
) is a real symmetric form.
Main result follows.
(5.6) Big Theorem (Completeness of characters). The C-irreducible characters of G
form an orthonormal basis of the space of class functions of G. Moreover,
(a) If : G GL(V ),

: G GL(V

) are irreducible representations of G aording


characters

,

then

) =
_
1 if ,

are isomorphic
0 otherwise
(b) Each class function of G can be expressed as a linear combinations of irreducible
characters of G.
Proof. In chapter 6.
(5.7) Corollary. Complex representations of nite groups are characterised by their char-
acters.
Proof. Have : G GL(V ) aording

. (G nite, F = C.) Complete reducibility (3.3) says
= m
1

1
. . .m
k

k
, where
j
is irreducible and m
j
0. Then m
j
=

j
) where

j
is aorded by
j
, since

= m
1

1
+. . . +m
k

k
and

j
) = m
1

1
+. . . +m
k

k
,

j
) =
m
j
, by (5.6)(a).
14
(5.8) Corollary (Irreducibility criterion). If is a complex representation of G aording

then irreducible

) = 1.
Proof. () Orthogonality.
() Assume

) = 1. (3.3) says

=

m
j

j
, for

j
irreducible, m
j
0. Then

m
2
j
= 1, so

=

j
for some j. Therefore

is irreducible.
(5.9) Theorem. If the irreducible complex representations of G,
1
, . . .,
k
, have dimensions
n
1
, . . ., n
k
, then [G[ =

i
n
2
i
.
(Recall end of chapter 4.)
Proof. Recall from (3.5),
reg
: G GL(CG), the regular representation of G, of dimension
[G[. Let
reg
be its character.
Claim.
reg
(1) = [G[ and
reg
(h) = 0 if h ,= 1.
Proof. Easy. Let G = g
1
, . . ., g
n
and take h G, h ,= 1. For 1 i n, hg
i
= g
j
,
some j ,= i, so i
th
row of [
reg
(h)]
B
has 0s in every place, except column j in
particular, the (i, i)
th
entry is 0 for all i. Hence
reg
(h) = tr [
reg
(h)]
B
= 0.
By claim,
reg
=

n
j

j
with n
j
=

j
(1) :
n
j
=
reg
,

j
) =
1
[G[

gG

reg
(g)

j
(g) =
1
[G[
[G[

j
(1) =

j
(1)

(5.10) Corollary. The number of irreducible characters of G (up to equivalence) equals k,


the number of conjugacy classes.
(5.11) Corollary. Elements g
1
, g
2
G are conjugate i

(g
1
) =

(g
2
) for all irreducible
characters of G.
Proof. () Characters are class functions.
() Let be the characteristic function of the class of g
1
. Then is a class function, so
can be written as a linear combination of the irreducible characters of G, by (5.6)(b).
Hence (g
2
) = (g
1
) = 1. So g
2
(
G
(g
1
).
Recall from (5.5) the inner product on ((G) and the real symmetric form , ) for characters.
(5.12) Denition. G nite, F = C. The character table of G is the k k matrix X =
[

i
(g
j
)] where

1
(= 1),

2
, . . .,

k
are the irreducible characters of G, and (
1
(= 1), (
2
, . . ., (
k
are the conjugacy classes, with g
j
(
j
.
I.e., the (i, j)
th
entry of X is

i
(g
j
).
Examples. C
2
= x : x
2
= 1)
1 x

1
1 1

2
1 1
C
3
= x : x
3
= 1)
1 x x
2

1
1 1 1

2
1
2

3
1
2

where = e
2i/3

3
.
G = D
6
= a, b : a
3
= b
2
= 1, bab
1
= a
1
)

= S
3
.
15
In (2.17) we found a complete set of non-isomorphic irreducible CG-modules: U
1
, U
2
, U
3
.
Let

i
=

Ui
, (1 i 3).
1 a, a
2
b, ab, a
2
b g
j

1
1 1 1

2
1 1 1

3
2 1 0

1
2
+1
2
+2
2
=6
6 3 2 |CG(gj)|
1 2 3 |C(gj)|
Orthogonality:
1 2
6
+
(1)(1)
3
= 0 (rows 2 & 3)
2
2
6
+
(1)
2
3
= 1 (row 3)
16
6. Proofs and Orthogonality
We want to prove (5.6), the Big Theorem. Well do this in two ways.
Proof 1 of (5.6)(a). Fix bases of V and V

. Write R(g), R

(g) for the matrices of (g),

(g) with respect to these, respectively.

) =
1
[G[

(g
1
)

(g) =
1
[G[

gG
1in

1jn
R

(g
1
)
ii
R(g)
jj
Let : V V

be linear, and dene


average
=

: V V

, v
1
[G[

gG

(g
1
)(g)v.
Then

is a G-homomorphism. For if h G,

(h
1
)

(h)(v) =
1
[G[

gG

_
(gh)
1
_

_
(gh)
_
(v) =
1
[G[

(g
1
)(g

)(v) =

(v).
Assume rst that ,

are not isomorphic. Schurs Lemma says



= 0 for any linear
: V V

.
Let =

having matrix E

(with respect to our basis), namely 0 everywhere except


1 in the (, )
th
place.
Then

= 0, so
1
[G[

gG
_
R

(g
1
)E

R(g)
_
ij
= 0.
Thus
1
[G[

gG
R

(g
1
)
i
R(g)
j
= 0 for all i, j.
With = i, = j,
1
[G[

gG
R

(g
1
)
ii
R(g)
jj
= 0. Sum over i, j and conclude

) = 0.
Now assume that ,

are isomorphic, so

=

. Take V = V

, =

. If : V V
is linear, then

Hom
G
(V, V ).
Now tr = tr

, as tr

=
1
[G[

tr
_
(g
1
)(g)
_
=
1
[G[

tr = tr .
By Schur,

= id for some C (depending on ). Now =
1
n
tr .
Let =

, so tr =

. Hence

=
1
n

id =
1
[G[

g
(g
1
)

(g).
In terms of matrices, take the (i, j)
th
entry:
1
[G[

g
R(g
1
)
i
R(g)
j
=
1
n

ij
,
and put = i, = j to get
1
[G[

g
R(g
1
)
ii
R(g)
jj
=
1
n

ij
.
Finally sum over i, j:

) = 1.
17
Before proving (b), lets prove column orthogonality, assuming (5.10).
(6.1) Theorem (column orthogonality).
k

i=1

i
(g
j
)

i
(g
l
) =
jl
[C
G
(g
j
)[.
This has an easy corollary:
(6.2) Corollary. [G[ =
k

i=1

2
i
(1).
Proof of (6.1).
ij
=

i
,

j
) =

l
1
[C
g
(g
l
)[

i
(g
l
)

j
(g
l
).
Consider the character table X = (

i
(g
j
)).
Then XD
1
X
t
= I
kk
, where D =
_
[C
G
(g
1
)[
.
.
.
[C
G
(g
k
)[
_
.
As X is a square matrix, it follows that D
1
X
t
is the inverse of X. So X
t
X = D.
Proof of (5.6)(b). List all the irreducible characters

1
, . . .,

l
of G. Its enough to show
that the orthogonal complement of span

1
, . . .,

l
in ((G) is 0.
To see this, assume f ((G) with f,

j
) = 0 for all irreducible

j
.
Let : G GL(V ) be irreducible aording

1
, . . .,

l
. Then f,

) = 0.
Consider
1
[G[

f(g)(g) : V V . This is a G-homomorphism, so as is irreducible


it must be id for some C (by Schur).
Now, n = tr
1
[G[

f(g)(g) =
1
[G[

f(g)

(g) = 0 = f,

).
So = 0. Hence

f(g)(g) = 0, the zero endomorphism on V , for all representations


. Take =
reg
, where
reg
(g) : e
1
e
g
(g G), the regular representation.
So

g
f(g)
reg
(g) : e
1

g
f(g)e
g
. It follows that

f(g)e
g
= 0.
Therefore f(g) = 0 for all g G. And so f = 0.
Various important corollaries follow from this:
(5.10) # irreducibles of G = # conjugacy classes
(6.1) column orthogonality
(6.2) [G[ =

2
i
(1) =

k
i=1
n
2
i
(5.11) irreducible

is constant on conjugacy classes i.e., g
1

G
g
2


(g
1
) =

(g
2
).
g, g
1
are G-conjugate

(g) R for all irreducible

as

(g
1
) =

(g).
18
Example. 6 3 2 |CG(gj )|
1 a b g
j

1
1 1 1

2
1 1 1

3
2 1 0 coming from operations on equilateral triangle
Column orthogonality:
3

i=1

i
(g
r
)

i
(g
s
).
r = 1, s = 2 : 1.1 + 1.1 + 2(1) = 0 r ,= s
r = 1, s = 3 : 1.1 + 1(1) + 2.0 = 0 r ,= s
r = 2, s = 2 : 1.1 + 1.1 + (1)(1) = 3 r = s, weight by [C
G
(g
r
)[
** Non-examinable section **
Proof 2 of (5.6)(a). (Uses starred material at the end of chapter 4.)
X irreducible G-space, V any G-space. V =

m
i=1
U
i
, with U
i
irreducible.
Then #U
j
isomorphic to X is independent of the decomposition. We wrote (V : X)
for this number, and in (4.7) we observed (V : X) = dim
C
Hom
G
(V, X) ().
Let : G GL(U) have character

Write U
G
= u U : (g)u = u g G, the
G-invariants of U.
Consider the map : U U, u
1
[G[

g
(g)u.
This is a projection onto U
G
(because its a G-homomorphism, and when restricted
to U
G
it acts as the identity there). Verify dimU
G
= tr =
1
|G|

g

U
(g) () (by
decomposing U and looking at bases).
Now choose U = Hom
C
(V, V

) with V, V

being G-spaces. G acts on U via g.(v) =

V
(g)
_

V
(g
1
)v
_
for U.
But Hom
G
(V, V

) =
_
Hom
C
(V, V

)
_
G
, so by (), dim
C
Hom
G
(V, V

) =
1
[G[

U
(g).
Finally, show

U
(g) =

V
(g
1
)

V
(g) see section on tensor products in chapter 9.
The orthogonality of the irreducible characters now follows from ().
** End of non-examinable section **
19
7. Permutation Representations
Preview was given in (3.7). Recall:
G nite, acting on nite set X = x
1
, . . ., x
n
.
CX = C-space, basis e
x1
, . . ., e
xn
of dimension [X[. CX =
_
j
a
j
e
xj
: a
j
C
_
.
corresponding permutation representation,
X
: G GL(CX), g (g), where (g) :
e
xj
e
gxj
, extended linearly. So
X
(g) :

xX
a
x
e
x

xX
a
x
e
gx
.

X
is the permutation representation corresponding to the action of G on X.
matrices representing
X
(g) with respect to the basis e
x

xX
are permutation ma-
trices: 0 everywhere except one 1 in each row and column, and ((g))
ij
= 1 precisely
when gx
j
= x
i
.
(7.1) Permutation character
X
is
X
(g) = [x
X
(g)[ = [x X : gx = x[.
(7.2)
X
always contains 1
G
. For: span(e
x1
+. . . +e
xn
) is a trivial G-subspace of CX with
G-invariant complement span(

a
x
e
x
:

a
x
= 0).
(7.3) Burnsides Lemma (Cauchy, Frobenius).
X
, 1) = # orbits of G on X.
Proof. If X = X
1
. . . X
l
, a disjoint union of orbits, then
X
=
X1
+. . . +
X
l
with
Xj
the permutation character of G on X
j
. So to prove the claim, its enough to show that
if G is transitive on X then
X
, 1) = 1.
So, assume G is transitive on X. Then

X
, 1) =
1
[G[

gG

X
(g)
=
1
[G[

_
(g, x) GX : gx = x
_

=
1
[G[

xX
[G
x
[ (G
x
= stabiliser of x)
=
1
[G[
[X[ [G
x
[ =
1
[G[
[G[ = 1

(7.4) Lemma. Let G act on sets X


1
, X
2
. Then G acts on X
1
X
2
via g(x
1
, x
2
) = (gx
1
, gx
2
).
The character
X1X2
=
X1

X2
and so
X1
,
X2
) = # orbits of G on X
1
X
2
.
Proof.
X1
,
X2
) =
X1

X2
, 1) =
X1X2
, 1) = # orbits of G on X
1
X
2
(by (7.3)).
(7.5) Let G act on X, [X[ > 2. Then G is 2-transitive on X if G has just two orbits on
X X, namely (x, x) : x X and (x
1
, x
2
) : x
i
X, x
1
,= x
2
.
(7.6) Lemma. Let G act on X, [X[ > 2. Then
X
= 1 +

with

irreducible G is
2-transitive on X.
Proof.
X
= m
1
1 +m
2

2
+. . . +m
l

l
with 1,

2
, . . .,

l
distinct irreducibles and m
i
Z
0
.
Then
X
,
X
) =

l
i=1
m
2
i
. Hence G is 2-transitive on X i l = 2, m
1
= m
2
= 1.
(7.7) S
n
acting on X
n
(see 1.6) is 2-transitive. Hence
Xn
= 1 +

with

irreducible of
degree n 1. Similarly for A
n
(n > 3)
20
(7.8) Example. G = S
4
.
Conjugacy classes correspond to dierent cycle types.
1 3 8 6 6 sizes
1 (12)(34) (123) (1234) (12) ccl reps
trivial

1
1 1 1 1 1
_
two linear characters
since S
4
/S

4
= C
2 sign

2
1 1 1 1 1

X4
1

3
3 1 0 1 1

2


4
3 1 0 1 1 product of the two above

5
d x y z w
Know: 24 = 1 + 1 + 9 + 9 +d
2
d = 2.
Column orthogonality: 1 + 1 3 3 + 2x = 0 x = 2
1 + 1 + 2y = 0 y = 1
1 1 3 + 3 + 2z = 0 z = 0
1 1 + 3 3 + 2w = 0 w = 0
Or:

reg
=

1
+

2
+ 3

3
+ 3

4
+ 2

5


5
=
1
2
(

reg

2
3

3
3

4
).
Or: can obtain

5
by observing S
4
/V
4

= S
3
and lifting characters see chapter 8.
(7.9) Example. G = S
5
.
1 15 20 24 10 20 30 |Cj|
1 (12)(34) (123) (12345) (12) (123)(45) (1234) gj
trivial

1
1 1 1 1 1 1 1
sign

2
1 1 1 1 1 1 1

X5
1

3
4 0 1 1 2 1 0

2


4
4 0 1 1 2 1 0

5
5 1 1 0 1 1 1

2


6
5 1 1 0 1 1 1

7
6 2 0 1 0 0 0
There are various methods to get

5
,

6
of degree 5.
One way is to note that if X = Syl
5
(G) then [X[ = 6 and one checks that

X
,

X
) = 2.
Therefore
X
1 is irreducible.
For

7
, rst

d
2
i
= 120 gives deg

7
= 6, and orthogonality for the remaining entries.
Or: let S
5
act on the set of
_
5
2
_
unordered pairs of elements of 1, 2, 3, 4, 5.

(
5
2
)
: 10 2 1 0 4 1 0

(
5
2
)
,

(
5
2
)
) = 3

(
5
2
)
, 1) = 1

(
5
2
)
,

3
) = 1
_


(
5
2
)
= 1 +

3
+
has degree 5 (and is actually

6
in the table).
See chapter 10 for the method of induced characters, and chapter 9 for symmetric and
alternating powers.
21
(7.10) Alternating groups.
Let g A
n
. Then [(
Sn
(g)[ = [S
n
: C
Sn
(g)[
An index 2 in Sn
[(
An
(g)[ = [A
n
: C
An
(g)[
but not necessarily equal: e.g., if = (123), then (
An
() = , but (
Sn
() = ,
1
.
We know [S
n
: A
n
[ = 2 and in fact:
(7.11) If g A
n
then (
Sn
(g) = (
An
(g) precisely when g commutes with some odd permuta-
tion; otherwise it breaks up into two classes of equal size. (In the latter case, precisely
when the disjoint cycle decomposition of g is a product of odd cycles of distinct lengths.)
Proof. See James & Liebeck 12.17
(7.12) G = A
4
. Write = e
2i/3
.
1 3 4 4 |Cj|
1 (12)(34) (123) (123)
1
gj
1G

1
1 1 1 1
X1

2
3 1 0 0

3
1 1
2

4
1 1
2

P
d
2
i
=12=1
2
+3
2
+?
2
+?
2
?=1
Final two linear characters are found via G/G

= G/V
4
= C
3
, by lifting see chapter 8.
For A
5
see Telemann chapter 11, or James & Liebeck 20.13 .
22
8. Normal Subgroups and Lifting Characters
(8.1) Lemma. Let N G, let : G/N GL(V ) be a representation of G/N. Then
: G
q
G/N

GL(V ) is a representation of G, where (g) = (gN) (and q is the
natural homomorphism). Moreover, is irreducible if is.
The corresponding characters satisfy

(g) =

(gN) for g G, and deg

= deg

. We
say that

lifts to

.
The lifting sending



is a bijection between
irreducibles of G/N irreducibles of G with N in the kernel
Proof. (See examples sheet 1, question 4.)
Note:

(g) = tr ((g)) = tr ( (gN)) =

(gN) g, and

(1) =

(N), so deg

= deg

.
Bijection. If

is a character of G/N and

is a lift to G then

(N) =

(1). Also, if
k N then

(k) =

(kN) =

(N) =

(1). So N ker

.
Now let

be a character of G with N ker

. Suppose : G GL(V ) aords



.
Dene : G/N GL(V ), gN (g) for g G. This is well-dened (as N ker

)
and is a homomorphism, hence a representation of G/N. If

is the character of
then

(gN) =

(g) for all g G.
Finally, check irreducibility is preserved.
Denition. The derived subgroup of G is G

= [a, b] : a, b G), where [a, b] = aba


1
b
1
is the commutator of a and b. (G

is a crude measure of how abelian a group is.)


(8.2) Lemma. G

is the unique minimal normal subgroup of G such that G/G

is abelian.
(I.e., G/N abelian G

N, and G/G

is abelian.)
G has precisely l = [G/G

[ representations of degree 1, all with kernel containing G

and obtained by lifting from G/G

.
Proof. G

G easy exercise.
Let N G. Let g, h G. Then g
1
h
1
gh N ghN = hgN (gN)(hN) =
(hN)(gN). So G

N G/N abelian. Since G

G, G/G

is an abelian group.
By (4.5), G/G

has exactly l irreducible characters,



1
, . . .,

l
, all of degree 1. The
lifts of these to G also have degree 1 and by (8.1) these are precisely the irreducible
characters

i
of G such that G

ker

i
.
But any linear character

of G is a homomorphism

: G C

, hence

(ghg
1
h
1
) =

(g)

(h)

(g
1
)

(h
1
) = 1.
Therefore G

ker

, so the

1
, . . .,

l
are all irreducible characters of G.
Examples. (i) Let G = S
n
. Show G

= A
n
. Thus G/G

= C
2
.
So S
n
must have exactly two linear characters.
23
(ii) G = A
4
.
1 (12)(34) (123) (123)
1
1
G
1 1 1 1

2
1 1
2

3
1 1
2

4
3 1 0 0
1 x x
2

1
1 1 1

2
1
2

3
1
2

( = e
2i/3
)
_

Let N = 1, (12)(34), (13)(24), (14)(23) G. In fact, N



= V
4
, NG, and G/N

= C
3
.
Also, G

= V
4
, so G/G

= C
3
.
(8.3) Lemma. G is not simple i

(g) =

(1) for some irreducible character

,= 1
G
and
1 ,= g G. Any normal subgroup of G is the intersection of kernels of some of the
irreducibles of G, N =

i irred
ker

i
.
Proof. If

(g) =

(1) for some non-principal character

(aorded by , say), then g ker
(by (5.3)). Therefore if g ,= 1 then 1 ,= ker G.
If 1 ,= N G, take an irreducible

of G/N (

,= 1
G/N
). Lift to get an irreducible

aorded by of G, then N ker G. Therefore

(g) =

(1) for g N.
In fact, if 1 ,= NG then N is the intersection of the kernels of the lifts of all irreducibles
of G/N. is clear. For : if g G N then gN ,= N, so

(gN) ,=

(N) for some
irreducible

of G/N, and then lifting

to

we have

(g) ,=

(1).
24
9. Dual Spaces and Tensor Products of Representations
Recall (5.5), (5.6): ((G) = C-space of class functions of G, dim
C
((G) = k, basis

1
, . . .

k
the irreducible characters of G.
(f
1
+f
2
)(g) = f
1
(g) +f
2
(g)
(f
1
f
2
)(g) = f
1
(g)f
2
(g)
involution (homomorphism of order 2) f f

where f

(g) = f(g
1
)
inner product , )
Duality
(9.1) Lemma. Let : G GL(V ) be a representation over F and let V

= Hom
F
(V, F),
the dual space of V .
Then V

is a G-space under

(g)(v) = ((g
1
)v), the dual representation of .
Its character is

(g) =

(g
1
).
Proof.

(g
1
)
_

(g
2
)
_
(v) =
_

(g
2
)
__
(g
1
1
)v
_
=
_
(g
1
2
)(g
1
1
)v
_
=
_
(g
1
g
2
)
1
(v)
_
=
_

(g
1
g
2
)
_
(v)
Character. Fix g G and let e
1
, . . ., e
n
be a basis of V of eigenvectors of (g), say
(g)e
j
=
j
e
j
. let
1
, . . .,
n
be the dual basis.
Then

(g)
j
=
1
j

j
, for
_

(g)
j
_
(e
i
) =
j
_
(g
1
)e
i
_
=
j

1
j
e
i
=
1
j

j
e
i
for all i.
Hence

(g) =

1
j
=

(g
1
).
(9.2) Denition. : G GL(V ) is self-dual if V

= V

(as an isomorphism of G-spaces).


Over F = C, this holds i

(g) =

(g
1
), and since this =

(g), it holds i

(g) R for
all g.
Example. All irreducible representations of S
n
are self-dual: the conjugacy classes are
determined by cycle types, so g, g
1
are always S
n
-conjugate. Not always true for A
n
: its
okay for A
5
, but not for A
7
see sheet 2, question 8.
Tensor Products
V and W, F-spaces, dimV = m, dimW = n. Fix bases v
1
, . . ., v
m
and w
1
, . . ., w
n
of V, W,
respectively. The tensor product space V W (or V
F
W) is an mn-dimensional F-space
with basis v
i
w
j
: 1 i m, 1 j n. Thus:
(a) V W =
_

1im
1jn

ij
v
i
w
j
:
ij
F
_
, with obvious addition and scalar multiplication.
(b) if v =

i
v
i
V , w =

j
w
j
W, dene v w =

i,j

i

j
(v
i
w
j
).
Note: not all elements of V W are of this form. Some are combinations, e.g. v
1
w
1
+v
2
w
2
,
which cannot be further simplied.
25
(9.3) Lemma. (i) For v V , w W, F, have (v) w = (v w) = v (w)
(ii) If x, x
1
, x
2
V and y, y
1
, y
2
W, then (x
1
+ x
2
) y = (x
1
y) + (x
2
y) and
x (y
1
+y
2
) = (x y
1
) + (x y
2
).
Hence V W V W, (v, w) v w is bilinear.
Proof. (i) v =

i
v
i
, w =

j
w
j
, then (v) w =

i,j
(
i
)
j
v
i
w
j
, and (v w) =

i,j

i

j
v
i
w
j
, and v (w) =

i,j

i
(
j
)v
i
w
j
.
All three are equal. (ii) is similar.
(9.4) Lemma. If e
1
, . . ., e
m
is a basis of V and f
1
, . . ., f
n
is a basis of W, then e
i
f
j
:
1 i m, 1 j n is a basis of V W.
Proof. Writing v
k
=

ik
e
i
, w
l
=

j

jl
f
j
, we have v
k
w
l
=

i,j

ik

jl
e
i
f
j
, hence
e
i
f
j
spans V W and since there are mn of them, they are a basis.
(9.5) Digression. (Tensor products of endomorphisms.) If : V V , : W W are
linear endomorphisms, dene : V W V W, v w (v) (w), and extend
linearly on a basis.
Example. Given bases / = e
1
, . . ., e
m
of V , and B = f
1
, . . ., f
n
of W, if []
A
= A
and []
B
= B, then ordering the basis / B lexicographically (i.e., e
1
f
1
, e
1
f
2
,. . .,
e
1
f
n
,e
2
f
1
,. . . ), we have
[ ]
AB
=
_

_
[a
11
B] [a
12
B] . . .
[a
21
B] [a
22
B] . . .
.
.
.
.
.
.
.
.
.
_

_
(9.6) Proposition. Let : G GL(V ),

: G GL(V

) be representations of G. Dene

: G GL(V V

) by
(

)(g) :

ij
v
i
w
j

ij
(g)v
i

(g)w
j
Then

is a representation of G, with character


(g) =

(g)

(g) for all


g G.
Hence the product of two characters of G is also a character of G. Note: example sheet 2,
question 2, says that if is irreducible and is degree 1, then

is irreducible. if

is
not of degree 1, then this is usually false, since

is usually reducible.
Proof. Clear that (

)(g) GL(V V

) for all g, and so

is a homomorphism
G GL(V V

).
Let g G. Let v
1
, . . ., v
m
be a basis of V of eigenvectors of (g), and w
1
, . . ., w
n
be a
basis of V

of eigenvectors of

(g). So (g)v
j
=
j
v
j
,

(g)w
j
=
j
w
j
.
Then (

)(g)(v
i
w
j
) = (g)v
i

(g)w
j
=
i
v
i

j
w
j
= (
i

j
)(v
i
w
j
).
So

(g) =

i,j

j
=
m

i=1

i
n

j=1

j
=

(g)

(g).
Take V = V

and dene V
2
= V V . Let :

ij
v
i
v
j

ij
v
j
v
i
, a linear
G-endomorphism of V
2
such that
2
= 1. Therefore, eigenvalues = 1.
26
(9.7) Denition. The symmetric square of V is S
2
V = x V
2
: (x) = x.
The exterior square (or anti-symmetric square, or wedge) of V is
2
V = x
V
2
: (x) = x.
(9.8) Lemma. S
2
V and
2
V are G-subspaces of V
2
, and V
2
= S
2
V
2
V .
S
2
V has a basis v
i
v
j
:= v
i
v
j
+v
j
v
i
, 1 i j n, so dimS
2
V =
1
2
n(n + 1).

2
V has a basis v
i
v
j
:= v
i
v
j
v
j
v
i
, 1 i j n, so dim
2
V =
1
2
n(n 1).
Proof. Elementary linear algebra.
To show V
2
is reducible, write x V
2
as x =
1
2
(x +(x))
. .
S
2
+
1
2
(x (x))
. .

2
.
(9.9) Lemma. If : G GL(V ) is a representation aording character

, then

2
=

S
+

where

S
(=S
2

) is the character of G on the subrepresentation on S


2
V , and

(=
2

) is the character of G on the subrepresentation on


2
V .
Moreover, for g G,

S
(g) =
1
2
_

2
(g) +

(g
2
)
_
and

(g) =
1
2
_

2
(g)

(g
2
)
_
.
Proof. Compute characters

S
,

. Fix g G. Let v
1
, . . ., v
m
be a basis of V of eigenvectors
of (g), say (g)v
i
=
i
v
i
. Then g v
i
v
j
=
i

j
v
i
v
j
and g v
i
v
j
=
i

j
v
i
v
j
.
Hence

S
(g) =

1ijn

j
and

(g) =

1i<jn

j
.
Now
_

(g)
_
2
=
_

i
_
2
=

2
i
+ 2

i<j

j
=

(g
2
) + 2

(g).
So,

(g) =
1
2
_

2
(g)

(g
2
)
_
, and so

S
(g) =
1
2
_

2
(g) +

(g
2
)
_
, as

2
=

S
+

.
Usual trick to nd characters: diagonalise and hope for the best!
Example. G = S
5
(again)
1 15 20 24 10 20 30 |Cj|
1 (12)(34) (123) (12345) (12) (123)(45) (1234) gj
1
G
=

1
1 1 1 1 1 1 1
sign =

2
1 1 1 1 1 1 1

= [x
[1,5]
(g)[1 =

3
4 0 1 1 2 1 0

2
=

4
4 0 1 1 2 1 0
S
2

3
=

5
5 1 1 0 1 1 1

2
=

6
5 1 1 0 1 1 1

=

7
6 2 0 1 0 0 0
We use (9.9) on

4
=

.
1 (12)(34) (123) (12345) (12) (123)(45) (1234)

2
(g) 16 0 1 1 4 1 0

(g
2
) 4 4 1 1 4 1 0

S
(g) 10 2 1 0 4 1 0

(g) 6 2 0 1 0 0 0
We have seen

S
already as
(
5
2
)
. Check inner product = 3; contains 1,

3
.
27
Characters of GH (cf. (4.5) for abelian groups)
(9.10) Proposition. If G, H are nite groups, with irreducible characters

1
, . . .,

k
and

1
, . . .,
l
respectively, then the irreducible characters of the direct product GH are
precisely

j
: 1 i k, 1 j l where

i

j
(g, h) =

i
(g)
j
(h).
Proof. If : G GL(V ) aording

and

: H GL(W) aording , then

:
G H GL(V W), (g, h) (g)

(h) is a representation of G H on V W
by (9.6). And

=

, also by (9.6).
Claim:

i

j
are distinct and irreducible, for:

j
,

s
)
GH
=
1
[GH[

(g,h)

j
(g, h)

s
(g, h)
=
_
1
[G[

i
(g)

r
(g)
__
1
[H[

j
(h)
s
(h)
_
=
ir

js
Complete set:

i,j

i

j
(1)
2
=

2
i
(1)

j

2
j
(1) = [G[ [H[ = [GH[.
Exercise. D
6
D
6
has 9 characters.
Digression: a general approach to tensor products
V, W, F-spaces (general F, even a non-commutative ring).
(9.11) Denition. V W is the F-space with a bilinear map t : V W T, (v, w)
v w =: t(v, w), such that any bilinear f : V W X (X any F-space) can be
factored through it:
I.e., there exists linear f

: T X such that f

t = f.
V W
t
T
f f

X
The triangle commutes
This is the universal property of the tensor product.
Claim. Such T exists and is unique up to isomorphism.
Existence. Take (huge) space M with basis (v, w) : v V, w W. Factor out the
subspace N generated by all the things you want to be zero, i.e. by
(v
1
+v
2
, w) (v
1
, w) (v
2
, w)
(v, w
1
+w
2
) (v, w
1
) (v, w
2
)
(v, w) (v, w), (v, w) (v, w)
for all v, v
1
, v
2
V , w, w
1
, w
2
W, F.
Dene t to be the map embedding V W M followed by the natural quotient map
V W
t
M/N
f f

X
Check t is bilinear (weve quotiented out the relevant properties to make it so). f

is
28
dened on our basis of M, (v, w) f(v, w), extended linearly. f

= 0 on all elements
of N, hence well-dened on M/N.
Uniqueness. V W T

T

Apply universal property with respect to T, T

.
Linear maps give isomorphism.

Henceforth, we think of V W as being generated by elements v w (v V, w W) and


satisfying
(v
1
+v
2
) w = v
1
w +v
2
w
v (w
1
+w
2
) = v w
1
+v w
2
(v w) = v w = v w
(9.12) Lemma. If e
1
, . . ., e
m
and f
1
, . . ., f
n
are bases of V , W respectively then e
i
f
j
:
1 i m, 1 j n is a basis of V W.
Proof. (Span.) Any v w can be expressed (hence so can any element of V W) as
v =

i
e
i
, w =

j

j
f
j
v w =

i,j

i

j
e
i
f
j
.
(Independence.) Find a linear functional sending e
i
f
j
to 1 and all the rest to 0.
For, take dual basis
i
,
j
to the above. Dene (v w) =
i
(v)
j
(w) and check
(e
i
f
j
) = 1, other = 0.
(9.13) Lemma. There is a natural (basis independent) isomorphism in each of the follow-
ing.
(i) V W

= W V
(ii) U (V W)

= (U V ) W
(iii) (U V ) W

= (U W) (V W)
Proof. (i) v w w v and extend linearly. Its well-dened: (v, w) w v is a
bilinear map V W W V . So by the universal property v w wv gives
a well-dened linear map.
(ii) u (v w) (u v) w and extend linearly. Its well-dened: x u U, then
(v, w) (u v) w is bilinear, so get v w (u v) w.
Varying u, (u, v w) (uv) w is a well-dened bilinear map U (V W)
(U V ) W. Hence, get linear map u (v w) (u v) w.
(iii) Similar. (See Telemann, chapter 6.)
(9.14) Lemma. Let dimV, dimW < . Then Hom(V, W)

= V

W naturally as G-spaces,
if V, W are both G-spaces.
Proof. The natural map V

W Hom(V, W), (, w) ( : v (v)w) is bilinear, so


w , extended linearly, is a linear map, V

W Hom(V, W).
Its bijective as it takes basis to basis:
i
f
j
(E
ji
: e
i
f
j
).
Returning to the proof of orthogonality at the end of chapter 6: the missing link was to
observe that U = Hom(V

, V )

= (V

V , hence

n
(g) =

(V

V
(g) =

V
(g
1
)

V
(g).
29
Symmetric and exterior powers
V an F-space, dimV = d, basis e
1
, . . ., e
d
, n N. Then V
n
= V . . . V (n times), of
dimension d
n
.
Note, S
n
acts on V
n
: for S
n
, (v
1
. . . v
n
) = v
(1)
. . . v
(n)
, and extend linearly
(place permutations).
The S
n
-action commutes with any G-action on V .
(9.15) Denition.
The symmetric powers, S
n
V = x V
n
: (x) = x for all S
n
.
The exterior powers,
n
V = x V
n
: (x) = sgn()x for all S
n
.
These are G-subspaces of V
n
, but if n > 2 then there are others obtained from the S
n
-action.
Exercises. Basis for S
n
V is
_
1
n!

Sn
v
i
(1)
. . . v
i
(n)
: 1 i
1
. . . i
n
d
_
.
Basis for
n
V is
_
1
n!

Sn
sgn()v
i
(1)
. . . v
i
(n)
: 1 i
1
< . . . < i
n
d
_
.
So dimS
n
V =
_
d +n 1
n
_
and dim
n
V =
_
d
n
_
.
(9.16) Denition. Let T
n
V = V
n
= V . . . V .
The tensor algebra of V is T(V ) =

n0
T
n
V , where T
0
V = 0 an F-space with
obvious addition and scalar multiplication.
There is a product: for x T
n
V , y T
m
V , get x.y := x y T
n+m
V , thus giving a
graded algebra (with product T
n
V T
m
V T
n+m
V ).
Finally, dene:
S(V ) = T(V )/(ideal generated by u v v u) the symmetric algebra,
(V ) = T(V )/(ideal generated by v v) the exterior algebra.
Character ring
((G) is a ring, so the sum and product of characters are class functions. This chapter veried
that they are in fact characters aorded by the sum and tensor product of their corresponding
representations.
(9.17) The Z-submodule of ((G) spanned by the irreducible characters of G is the character
ring of G, written R(G).
Elements of R(G) are called
_
_
_
dierence
generalised
virtual
_
_
_
characters. R(G) : =

irred
n

, n Z.
R(G) is a commutative ring, and any generalised character is a dierence of two characters.
30
I.e., = , , characters, where =

n0
n

, =

n<0
n

.
The

i
form a Z-basis for R(G), as free Z-module.
Suppose R(G) and , ) = 1. Then (1) > 0 implies is the character of an irreducible
representation of G.
Proof. Let the irreducible characters of G be

1
, . . .,

k
. Then =

n
i

i
. So , ) =

n
2
i
= 1, so exactly one n
i
= 1. But (1) > 0, so one n
i
= 1 and the rest are 0.
Henceforth we dont distinguish between a character and its negative, and we often study
generalised characters of norm 1 (, ) = 1) rather than just irreducible characters.
31
10. Induction and Restriction
Throughout, H G.
(10.1) Denition (Restriction). Let : G GL(V ) be a representation aording

.
Can think of V as a H-space by restricting attention to h H.
Get Res
G
H
: H GL(V ), the restriction of to H. (Also written [
H
or
H
.)
It aords the character Res
G
H

=

[
H
=

H
.
(10.2) Lemma. If is any non-zero character of H, then there exists an irreducible char-
acter

of G such that
Res
G
H

is a constituent of Res
G
H

Res
G
H

, ) , = 0
_
_
_
3 ways of saying the same thing
Proof. List the irreducible characters of G:

1
, . . .

k
. Recall

reg
from (5.9).
0 ,=
[G[
[H[
(1) =

reg
[
H
, )
H
=

deg

i
[
H
, )
H
Therefore

i
[
H
, ) , = 0 for some i.
(10.3) Lemma. Let

be an irreducible character of G, and let Res
G
H

=

i
c
i

i
with

i
irreducible characters of H, where c
i
Z
0
.
Then

c
2
i
[G : H[, with equality i

(g) = 0 for all g G H.
Proof.

c
2
i
= Res
G
H

, Res
G
H

)
H
=
1
[H[

hH
[

(h)[
2
.
But 1 =

)
G
=
1
[G[

gG
[

(g)[
2
=
1
[G[
_

hH
[

(h)[
2
+

gG\H
[

(g)[
2
_
=
[H[
[G[

c
2
i
+
1
[G[

gG\H
[

(g)[
2
. .
0, and = 0

(g) = 0 g G H
Therefore

c
2
i
[G : H[, with equality i

(g) = 0 for all g G H.
Example. G = S
5
, H = A
5
,
i
= Res
G
H

i
.
=0 somewhere outside H

(g)=0 gS5\A5

deg

i
1 1 4 4 5 5 6
/ / / /
deg
i
1 4 5 3 3
..
general fact about normal subgroups:
splits into constituents of equal degree
(Cliords Theorem)
32
(10.4) Denition (Induction). If is a class function of H, dene

G
= Ind
G
H
(g) =
1
[H[

xG

(x
1
gx), where

=
_
(y) y H
0 y / H
.
(10.5) Lemma. If is a class function of H, then Ind
G
H
is a class function of G, and
Ind
G
H
(1) = [G : H[ (1).
Proof. Clear, noting that Ind
G
H
(1) =
1
[H[

xG

(1) = [G : H[ (1).
Let n = [G : H[. Let t
1
= 1, t
2
, . . ., t
n
be a left transversal of H in G (i.e., a complete set
of coset representatives), so t
1
H = H, t
2
H, . . ., t
n
H are precisely the left cosets of H in G.
(10.6) Lemma. Given a transversal as above, Ind
G
H
(g) =
n

i=1

(t
1
i
gt
i
).
Proof. For h H,

_
(t
i
h)
1
g(t
i
h)
_
=

(t
1
i
gt
i
), as is a class function of H.
(10.7) Theorem (Frobenius Reciprocity). a class function on H, a class function
on G. Then
Res
G
H
, )
H
= , Ind
G
H
)
G
.
Proof. ,
G
)
G
=
1
[G[

gG
(g)
G
(g)
=
1
[G[ [H[

g,xG
(g)

(x
1
gx)
=
1
[G[ [H[

x,y
(y)

(y) (put y = x
1
gx)
=
1
[H[

yG
(y)

(y) (independent of x)
=
1
[H[

yH
(y)(y)
=
H
, )
H
(10.8) Corollary. If is a character of H then Ind
G
H
is a character of G.
Proof. Let

be an irreducible character of G.
By (10.7), Ind
G
H
,

)
G
= , Res
G
H

) Z
0
, since , Res
G
H

are characters.
Hence Ind
G
H
is a linear combination of irreducible characters, with positive coecients,
hence a character.
(10.9) Lemma. Let be a character (or even a class function) of H G and let g G.
Let (
G
(g) H =

m
i=1
(
H
(x
i
) (disjoint union), where x
i
are representatives of the m
H-conjugacy classes of elements of H conjugate to g.
Then, for m = 0, Ind
G
H
(g) = 0, and for m 1, Ind
G
H
(g) = [C
G
(g)[
m

i=1
(x
i
)
[C
H
(x
i
)[
.
33
Diversion. Let H, K G. A double coset of H and K in G is a set of the form
HxK = hxk : h H, k K for some x G.
Facts. Two double cosets are either disjoint or equal, and
[HxK[ =
[H[ [K[
[H xKx
1
[
=
[H[ [K[
[xHx
1
K[
.
Proof of (10.9). If m = 0 then x G : x
1
gx H = , and Ind
G
H
(g) = 0 by denition.
Let m > 0. Let X
i
= x G : x
1
gx H and is conjugate in H to x
i
, for 1 i m.
The X
i
are pairwise disjoint and their union is x G : x
1
gx H. By denition,
Ind
G
H
(g) =
1
[H[

xG

(x
1
gx) =
1
[H[
m

i=1

xXi
(x
1
gx)
=
1
[H[
m

i=1

xXi
(x
i
) =
m

i=1
[X
i
[
[H[
(x
i
).
We need to calculate
[X
i
[
[H[
.
Fix 1 i m and choose g
i
G such that g
1
i
gg
i
= x
i
. So for all c C
G
(g) and
h H we have
(cg
i
h)
1
g(cg
i
h) = h
1
g
1
i
c
1
gcg
i
h = h
1
g
1
i
gg
i
h = h
1
x
i
h,
i.e. cg
i
h X
i
, and hence C
G
(g)g
i
H X
i
.
Conversely, for x X
i
, we have x
1
gx = h
1
x
i
h = h
1
(g
1
i
gg
i
)h for some h H. So
xh
1
g
1
i
C
G
(g), and hence x C
G
(g)g
i
h C
G
(g)g
i
H.
Thus C
G
(g)g
i
H = X
i
, so [X
i
[ = [C
G
(g)g
i
H[ =
[C
G
(g)[ [H[
[H g
1
i
C
G
(g)g
i
[
.
But g
1
i
C
G
(g)g
i
= C
G
(g
1
i
gg
i
) = C
G
(x
i
).
So [X
i
[ = [H : H C
G
(x
i
)[ [C
G
(g)[ = [H : C
H
(x
i
)[ [C
G
(g)[.
So
[X
i
[
[H[
=
[C
G
(g)[
[C
H
(x
i
)[
, giving the result.
(10.10) Lemma. If = 1
H
, the principal character of H, then Ind
G
H
1
H
=
X
, the permu-
tation character of G on the set X of left cosets of H in G.
Proof. Ind
G
H
1
H
(g) =


1
H
(t
1
i
gt
i
) (where the t
i
form a transversal)
=

i : t
1
i
gt
i
H

i : g t
i
Ht
1
i

stabiliser in G of the point t


i
H X
= [x
X
(g)[ =
X
(see (7.1))

34
Remark. Recalling (7.3):

X
, 1
G
)
G
=
(10.10)
Ind
G
H
1
H
, 1
G
)
G
=
(10.7)
1
H
, Res
G
H
1
G
)
H
= 1
H
, 1
H
)
H
= 1.
Examples. (a) Recall (7.9), G = S
5
acting on X = the set of Sylow 5-subgroups of G.

X
= Ind
G
H
1
H
, where H = (12345), (2354)). Note [H[ = 20.
H ccls 1 (12345) (2354) (2453) (25)(34)
size 1 4 5 5 5
Gccls 1 (12)(34) (123) (12345) (12) (123)(45) (1234)
size 1 15 20 24 10 20 30

X
(2354) =
120
30
(
5
20
+
5
20
) = 4(
1
4
+
1
4
) = 2

X
((25)(34)) =
120
15
(
5
20
) = 8(
1
4
) = 2. (All elements (25)(34) are conjugate in H.)
(b) Recall (2.17) and (7.8). H = C
4
= (1234)) G = S
4
, index 6.
Character of induced representation Ind
S4
C4
(), where is faithful 1-dimensional
representation of C
4
. If ((1234)) = i then character of is:
1 (1234) (13)(24) (1432)

1 i 1 i
Induced representations:
size 1 6 8 3 6
ccls 1 (12) (123) (12)(34) (1234)
Ind
S4
C4
() 6 0 0 2 0
For (12)(34), only one of 3 elements in S
4
that its conjugate to lies in H. So
Ind
G
H
() = 8(
1
4
) = 2.
(1234) is conjugate to 6 elements of S
4
, of which 2 are in C
4
(viz. (1234), (1432)).
So Ind
G
H
() = 4(
i
4

i
4
) = 0.
Induced modules
H G, index n. t
1
= 1, t
2
, . . ., t
n
a transversal i.e. H, t
2
H, . . ., t
n
H are the left cosets of
H in G. Let W be an H-space.
(10.11) Denition. Let V = W t
2
W . . . t
n
W, where t
i
W = t
i
w : w W.
(Essentially tensored group algebra with W.)
So dimV = ndimW and we write V = Ind
G
H
W.
G-action. g G, for all i, there exists a unique j with t
1
j
gt
i
H (namely t
j
H is the unique
coset which contains gt
i
).
Dene g(t
i
w) = t
j
_
(t
1
j
gt
i
)w
_
. (Drop the s, so t
i
w := t
i
w.) Check this is a G-action:
g
1
(g
2
t
i
w)
. .
= g
1
_
t
j
(t
1
j
g
2
t
i
)w
_
( unique j s.t. g
2
t
1
H = t
j
H)
= t

_
(t
1

g
1
t
j
)
. .
(t
1
j
g
2
t
i
)w
_
( unique s.t. g
1
t
j
H t

H)
= t

_
t
1

(g
1
g
2
)t
i
_
w
= (g
1
g
2
)(t
i
w)
35
is unique with (g
1
g
2
)t
i
H t

H.
It has the right character (still dropping the ) g : t
i
w t
j
(t
1
j
gt
i
. .
W
)w.
So the contribution to the character is 0 unless j = i, i.e. unless t
1
i
gt
i
H, then it
contributes (t
1
i
gt
i
), i.e. Ind
G
H
(g) =

n
i=1

(t
1
i
gt
i
), thus agreeing with (10.6).
Example. Module-theoretic version of (10.10) states: Ind
G
H
(C) = CX, where X = G/H.
In particular, Ind
G
1
(C) =
reg
.
Remarks (non-examinable). (1) There is also a Frobenius reciprocity for modules: for
W a H-space, V a G-space, Hom
H
(W, Res
G
H
V )

= Hom
G
(Ind
G
H
W, V ) naturally,
as vector spaces.
This is an example of a Nakayama relation. See Telemann 15.9 works over
general elds.
(2) Tensor products of modules over rings. In (10.11), V = FG
FH
W.
Replace FG by R, FH by S, and try to generalise. In general, given rings R, S,
and modules U an (R, S)-bimodule and W a left S-module, then U W is a left
R-module with balanced map t : U W U W such that any balanced map
f : U W X, any left R-module X can be factored through t.
U W
t
U W
f unique module homomorphismf

X
Balanced means f(u
1
+u
2
, w) = f(u
1
, w) +f(u
2
, w)
f(u, w
1
+w
2
) = f(u, w
1
) +f(u, w
2
)
f(u, w) = f(u, w) (for all S)
Then Ind
G
H
W = FG W is now a well-dened FG-module, since W is a left
FH-module, FG is (FG, FH)-bimodule. (Alperin-Bell.)
36
11. Frobenius Groups
(11.1) Frobenius Theorem (1891). G a transitive permutation group on a set X, [X[ =
n. Assume that each non-identity element of G xes at most one element of X. Then
K = 1 g G : g ,= for all X
is a normal subgroup of G of order n.
Proof. (Suzuki, Collins (book).) Required to prove K G.
Let H = G

(stabiliser of X), so conjugates of H are the stabilisers of single


elements of X, as G
g
= gG

g
1
. No two conjugates can share a non-identity element
(hypothesis).
So H has n distinct conjugates and G has n([H[ 1) elements that x exactly one
element of X. But [G[ = [X[ [H[ = n[H[. (X and G/H are isomorphic G-sets, as the
action is transitive), hence [K[ = [G[ n([H[ 1) = n.
Let 1 ,= h H. Suppose h = gh

g
1
, some g G, h

H. Then h lies in H = G

and
gHg
1
= G
g
. By hypothesis, g = , hence g H. So conjugacy class in G of h is
precisely the conjugacy class in H of h.
Similarly, if g C
G
(h) then h = ghg
1
G
g
hence g H, i.e. C
G
(h) = C
H
(h).
Every element of G lies either in K or in one of the n stabilisers, each of which is
conjugate to H. So every element of G K is conjugate with a non-1 element of H. So
1, h
2
, . . ., h
t
. .
, y
1
, . . ., y
u
. .

reps of H-ccls reps of ccls of G comprising K \ {1}


is a set of conjugacy class representatives for G.
Problem. To show K G.
Take = 1
G
, 1
H
=
1
,
2
, . . .,
t
irreducible characters of H. Fix some 1 i t.
Then if g G,
Ind
G
H

i
(g) =
_
_
_
[G : H[
i
(1) = n
i
(1) g = 1

i
(h
j
) g = h
j
(2 j t)
0 g = y
k
(1 k u)

#
C
G
(h
j
) = C
H
(h
j
) & (10.9)
Fix some 2 i t and put
i
=
G
i

i
(1)
G
1
+
i
(1)
1
R(G), by (9.16).
Values for 2 j t, 1 k u:
1 h
j
y
k

G
i
n
i
(1)
i
(h
j
) 0

i
(1)
G
1
n
i
(1)
i
(1) 0

i
(1)
1

i
(1)
i
(1)
i
(1)

i

i
(1)
i
(h
j
)
i
(1)
37

i
,
i
) =
1
[G[

gG
[
i
(g)[
2
=
1
[G[
_

gK
[
i
(g)[
2
+

1=gG
[
i
(g)[
2
_
=
1
[G[
_
n
2
i
(1) +n

1=hH
[
i
(h)[
2
_
=
1
[H[

[
i
(h)[
2
=
i
,
i
)
= 1 (row orthogonality of irreducible H-characters)
By (9.17) either
i
or
i
is an irreducible character of G, since
i
(1) > 0, it is
i
. Let
=

t
i=1

i
(1)
i
. Column orthogonality (h) =

t
i=1

i
(1)
i
(h) = 0 (1 ,= h H)
and for any y K, (y) =

2
i
(1) = [H[.
So (g) =
_
[H[ if g K
0 if g / K
Therefore K = g G : (g) = (1)
(5.3)
G.
(11.2) Denition. A Frobenius group is a group G having a subgroup H such that
H H
g
= 1 for all g H. H is a Frobenius complement.
(11.3) Any nite Frobenius group satises the hypothesis of (11.1). The normal subgroup
K is the Frobenius kernel of G.
If G is Frobenius and H a complement then the action of G on G/H is faithful and tran-
sitive. If 1 ,= g G xes xH and yH then g xHx
1
yHy
1
, which implies that
H (y
1
x)H(y
1
x)
1
,= 1, and so xH = yH.
Remarks. (i) Thompson (thesis, 1959) worked on the structure of Frobenius groups e.g.
showed that K is nilpotent (i.e., K is the direct product of its Sylow subgroups).
(ii) There is no proof of (11.1) known in which character theory is not used.
(iii*) Show that G = K H, semi-direct product.
38
12. Mackey Theory
This describes restriction to a subgroup K G of an induced representation W. K, H are
unrelated but usually we take K = H, in which case we can tell when Ind
G
H
W is irreducible.
Special case: W = 1 (trivial H-space, dim 1). Then by (10.10) Ind
G
H
1 = permutation
representation of G on X = G/H (coset action on the set of left cosets of H in G).
Recall. If G is transitive on a set X and H = G

( X) then the action of G on X is


isomorphic to the action on G/H, viz:
(12.1) g.
..
X
gH
..
G/H
is a well-dened bijection and commutes with G-actions.
I.e., x(g) = (xg) x(gH) = (xg)H.
Consider the action of G on G/H and restriction to some K G. G/H splits into K-orbits;
these correspond to double cosets KgH = kgh : k K, h H. The K-orbit containing
gH contains precisely all kgH (k K).
(12.2) Denition. K G/H is the set of double cosets KgH.
Note [K G/H[ =
G/K
,
G/H
) see (7.4). Clearly G
gH
= gHg
1
. Therefore K
gH
=
gHg
1
K. So by (12.1) the action of K on the orbit containing gH is isomorphic to the
action of K on K/(gHg
1
K).
(12.3) Proposition. Res
G
K
Ind
G
H
1 =

gK\G/H
Ind
K
gHg
1
K
1,
summed over set of representatives of double cosets.
Now choose g
1
, . . ., g
r
such that G =

Kg
i
H. Write H
g
= gHg
1
K K. Let W be
an H-space, and write W
g
for the H
g
-space with the same underlying vector space as W of
vectors, but with H
g
-action from
g
(x) = (g
1
xg
. .
H
) for x gHg
1
.
We will prove:
(12.4) Theorem (Mackeys Restriction Formula). Res
G
K
Ind
G
H
W =

gK\G/H
Ind
K
Hg
W
g
.
In terms of characters:
(12.5) Theorem. If ((H), then Res
G
K
Ind
G
H
=

gK\G/H
Ind
K
Hg

g
, where
g
is the class
function on H
g
given by
g
(x) = (x
g
1
).
The most useful form for applications is:
(12.6) Corollary (Mackeys Irreducibility Criterion). H G, W and H-space. Then
V = Ind
G
H
W is irreducible i
(i) W is irreducible, and
(ii) for each g G H, the two (gHg
1
H)-spaces W
g
and Res
H
Hg
W have no irre-
ducible constituents in common. (We say they are disjoint.)
39
Proof of Corollary. Take K = H in (12.4), so H
g
= gHg
1
H. Assume W is irreducible
with character .
Ind
G
H
, Ind
G
H
) =
(F.R.)
, Res
G
H
Ind
G
H
)
=
(12.5)

gH\G/H
, Ind
H
Hg

g
)
H
=
(F.R.)

gH\G/H
Res
H
Hg
,
g
)
Hg
= 1 +

gH\G/H
g/ H
d
g
where d
g
= Res
H
Hg
,
g
)
Hg
So to get irreducibility we need all the d
g
= 0.
(12.7) Corollary. If H G, assume is an irreducible character of H. Then Ind
G
H
is
irreducible i is distinct from all its conjugates
g
for g G H, where
g
(h) =
(h
g
1
) = (g
1
hg).
Proof. Take K = H, so H
g
= gHg
1
H = H for all g (since H G).
g
is the character
of H conjugate to , so Res
H
Hg
= and the
g
are just the conjugates of .
Proof of (12.4). Write V = Ind
G
H
W. Fix g G, so KgH K G/H. Observe V is a
direct sum of images of the form xW (ocially x W, recall), with x running over
representatives of left cosets of H in G (see (10.11)). Collect together the images xW
with x KgH (as in (12.3)) and dene V (g) =

xKgH
xW.
Now V (g) is a K-space and Res
G
K
V =

g reps of
K\G/H
V (g).
We have to prove V (g) = Ind
K
Hg
W
g
, as K-spaces. The subgroup of K consisting of the
elements x with xgW = gW is H
g
= gHg
1
K (see (12.2)), and V (g) =

xK\Hg
x(gW).
Hence V (g)

= Ind
K
Hg
(gW).
Finally W
g

= gW as K-spaces, as the map w gw is an isomorphism. Hence the
assertion.
Examples. (a) Give a direct proof of (12.3). Hint. Write G =
_
gi reps of
K\G/H
Kg
i
H ,(1 i r).
Let H
gi
have transversal k
i r1
, . . ., k
i ri
in K. Then k
ij
g
i
: 1 i r, 1 j r
i

is a transversal of K in G. Then compute Ind


G
H
(k).
(b) (Examples sheet 3, question 4.) C
n
D
2n
= x, y : x
n
= y
2
= 1, y
1
xy = x
1
).
Mackey says that for any 1-dimensional representation of C
n
, the 2-dimensional
representation Ind
D2n
Cn
is irreducible i is not isomorphic to
y
.
Now y
1
xy = x
1
, so this says that if (x) =
i
(
n
),
g
is the representation

g
(x) =
i
. So for 0 < i < n/2 (i.e. when e
2ik/n
,= e
2ik/n
) we get a
2-dimensional irreducible representation of D
2n
this way.
40
13. Integrality
(13.1) Denition. a C is an algebraic integer if it is a root of a monic polynomial
in Z[X]. Equivalently, the subring Z[a] = f(a) : f(x) Z[X] of C is a nitely-generated
Z-module.
Fact 1. The algebraic integers form a subring of C. (James & Liebeck 22.3)
Fact 2. If a C is both an algebraic integer and a rational number then a Z. (James &
Liebeck 22.5)
Fact 3. Any subring S of C which is nitely generated as a Z-module consists of algebraic
integers.
If s
1
, . . ., s
n
are generators of S as a Z-module, let a S. Then for all i, there exist
a
ij
Z with as
i
=

j
a
ij
s
j
. Put A = (a
ij
). Then Av = av, where v = (s
1
, . . ., s
n
)
t
, so
a is a root of the characteristic polynomial of A. Therefore, its an algebraic integer.
(13.2) Proposition. If

is a character of G and g G then

(g) is an algebraic integer.
Corollary. There are no entries in the character table of any nite group which are rational
but not integers. (Fact 2.)
Proof of (13.2).

(g) is the sum of n
th
roots of 1 (n = [g[). Each root of unity is an
algebraic integer, and any sum of algebraic integers is an algebraic integer. (Fact 1.)
Recall from (2.4) the group algebra CG =

g
g :
g
C of a nite group G, the C-space
with basis the elements of G. It is also a ring.
List (
1
= 1, (
2
, . . ., (
k
, the G-conjugacy classes. Dene the class sums, C
j
=

gCj
g CG.
Z(CG) is the centre of CG (not the same as CZ(G)).
(13.3) Proposition. C
1
, . . ., C
k
is a basis of Z(CG). There exist non-negative integers a
ijl
(1 i, j, l k) with C
i
C
j
=

a
ijl
C
l
. These are the structure constants for Z(CG).
E.g., 1, (12) + (13) + (23), (123) + (132) form a basis of Z(CS
3
).
Proof. gC
j
g
1
= C
j
, so C
j
Z(CG). Clearly the C
j
are linearly independent (because the
conjugacy classes are pairwise disjoint).
Now suppose z Z(CG), z =

gG

g
g. Then for all h G we have
h
1
gh
=
g
,
so the function g
g
is constant on G-conjugacy classes. Writing
g
=
i
(g (
i
),
then z =

j
C
j
.
Finally Z(CG) is a C-algebra (vector space over C with ring multiplication), so C
i
C
j
=

k
l=1
a
ijl
C
l
, as the C
j
span. We claim that a
ijl
Z
0
.
For: x g
l
(
l
, then a
ijl
= #(x, y) (
i
(
j
: xy = g
l
Z
0
.
(13.4) Denition. Let : G GL(V ) be an irreducible representation over C aording

. Extend by linearity to : CG EndV , an algebra homomorphism. Such a


homomorphism of algebras, CG = A End V is a representation of A.
41
Let z Z(CG). Then (z) commutes with all (g) (g G), so by Schurs Lemma (g) =
z
I
for some
z
C. Consider the algebra homomorphism w = w : Z(CG) C, z
z
.
Then (C
i
) = w(C
i
)I, so

(1)w(C
i
) =

gCi

(g) = [(
i
[

(g
i
) (g
i
a representative of (
i
).
Therefore w(C
i
) =

(g
i
)

(1)
[(
i
[.
(13.5) Lemma. The values of w(C
i
) =

(g
i
)

(1)
[(
i
[ are algebraic integers.
Proof. Since w is an algebra homomorphism, have w(C
i
)w(C
j
) =

k
l=1
a
ijl
w(C
l
), with
a
ijl
Z
0
. Thus the span w(C
i
) : 1 i k is a subring of C, so by Fact 3 consists
of algebraic integers.
Example. Show that a
ijl
= #(x, y) (
i
(
j
: xy = g
l
can be obtained from the character
table. In fact,
a
ijl
=
[G[
[C
G
(g
i
)[ [C
G
(g
j
)[
k

s=1

s
(g
i
)

s
(g
j
)

s
(g
1
l
)

s
(1)
.
Hint: use column orthogonality. (See James & Liebeck 30.4.)
(13.6) Theorem. The degree of any irreducible character of G divides [G[.
I.e.,

i
(1)

[G[ (1 i k).
Proof. Given irreducible

. (Standard trick: show [G[/

(1) N.)
[G[

(1)
=
1

(1)

gG

(g)

(g
1
)
=
1

(1)
k

i=1
[(
i
[

(g
i
)

(g
1
i
)
=
k

i=1
[(
i
[

(g
i
)

(1)

(g
1
i
)
Now
[(
i
[

(g
i
)

(1)
is an algebraic integer by (13.5), and

(g
1
i
) is a sum of roots of unity,
so is an algebraic integer by (13.2)
Thus
[G[

(1)
is an algebraic integer, and since its clearly rational, it is an integer.
Examples. (a) If G is a p-group then

(1) is a p-power (

irreducible). If [G[ = p
2
then

(1) = 1 (hence G is abelian).


(b) No simple group has an irreducible character of degree 2 (see James & Liebeck
22.13).
(c*) In fact, if

is irreducible then

(1) divides [G[/[Z[ (Burnside).
(d) G = S
n
: every prime p dividing the degree of an irreducible character of G also
divides n!. Hence p n.
42
(13.7) Theorem (Burnside). If

is irreducible, then

(1) divides
[G[
[Z[
.
Proof. Let : G GL(V ) be a representation with character

. For any m, consider

m
:
m
: G
m
GL(

m
V ).
Now ker
m
contains the subgroup Z

m
= (g
1
, . . ., g
m
) Z
m
: g
1
g
m
= 1.
If is irreducible then so is
m
by (9.11), and dim
m
= (dim)
m
[G
m
/Z

m
[ =
[G[
m
[Z[
m1
.
This is true for any m, so dim divides
[G[
[Z[
. (Check via prime factorisation.)
43
14. Burnsides p
a
q
b
Theorem
(14.1) Theorem (Burnside, 1904). p, q primes. Let [G[ = p
a
q
b
where a, b Z
0
, with
a +b 2. Then G is not simple.
Remarks. (1) In fact, even more is true: G is soluble. That is, there exists a chain
G = G
0
G
1
G
r
= 1 such that G
i
/G
i+1
is abelian for all i.
(2) The result is best possible: A
5
is simple, and 60 = 2
2
.3.5.
(3) If either a or b is 0 then [G[ = p-power and we know Z(G) ,= 1. Then there is
g Z, [g[ = p and g) G, with g) , = 1 or G.
(14.2) Proposition.

an irreducible C-character of G, ( a G-conjugacy class, g G such
that (

(1), [([) = 1. Then [

(g)[ =

(1) or 0.
Proof. There are a, b Z
0
such that a

(1)+b[([ = 1. Dene = a

(g)+
b

(g)

(1)
[([ =

(g)

(1)
.
Then is an algebraic integer, so the assertion follows from:
(14.3) Lemma. Assume =
1
m
m

i=1

i
is an algebraic integer with
n
j
= 1 for all j, some n.
Then [[ = 1.
For (14.2), we take n = [g[, m =

(1).
Proof (non-examinable). Assume [[ , = 0. Now F = Q() where = e
2i/n
and

j
F for all j.
Let ( = Gal(F/Q). Observe F :

= for all ( = F
G
= Q. (Result from
Galois Theory.)
Consider the norm N() of , namely the product of all the Galois conjugates

( (). The norm Q because its xed by all of (. Its an algebraic integer (all
Galois group conjugates of an algebraic integer are algebraic integers). Hence N() Z.
But N() =

is a product of expressions

roots of 1
m
C if absolute value 1.
Hence the norm must be 1, hence [[ = 1.
(14.4) Theorem. If in a nite group G the number of elements in a conjugacy class ( , = 1
is a p-power, then G is not non-abelian simple.
Remark. This implies (14.1). Assume a > 0, b > 0. Let Q Syl
q
(G). Then Z(Q) ,= 1, so
choose 1 ,= g Z(Q). So C
G
(g) Q. Therefore [((g)[ = [G : C
G
(g)[ = p
r
(some r).
Hence if p
r
= 1 then g Z(G). Therefore Z(G) ,= 1 (so not simple). If p
r
then G is
not simple (by (14.4)).
Proof of (14.4). Assume that G is non-abelian simple, and let 1 ,= g G with [(
G
(g)[ = p
r
.
By column orthogonality, 0 =

irred
of G

(1)

(g) ()
44
G is non-abelian simple, so [

(g)[ , =

(1) for any irreducible

,= 1. By (14.2), for any
irreducible character

,= 1 of G, we have p[

(1) or

(g) = 0.
Deleting zero terms in (), 0 = 1 +p

irred
p|(1)

(1)
p

(g).
Thus 1/p is an algebraic integer, since 1/p Q, hence 1/p Z. Contradiction.
Remarks. (a) In 1911, Burnside conjectured that if [G[ is odd then G is not non-abelian
simple. Only proved in 1963 by Feit & Thompson, a result which began the
Classication of Finite Simple Groups. The Classication only ended in 2005.
(b) A group-theoretic proof given only in 1972 (H. Bender)
45
15. Representations of Topological Groups
(15.1) A topological group is a group which is also a topological space such that the group
operations G G G, (h, g) hg and G G, g g
1
are continuous. It is compact if
it is so as a topological space.
(15.2) Basic examples. (a) GL
n
(R), GL
n
(C) are open subspaces of R
n
2
or C
n
2
.
(b) G nite, discrete topological. Also compact.
(c) G = S
1
= U(1) = g C : [g[ = 1.
(d) O(n) = A GL
n
(R) : AA
t
= I orthogonal group.
Compact: set of orthonormal bases for R
n
= (e
1
, . . ., e
n
) R
n
. . . R
n
:
e
i
, e
j
) =
ij
.
U(n) = A GL
n
(C) : AA
t
= I unitary group.
Compact: A U(n) i its columns are orthonormal.
Note: O(1) = Z/2, SO(1) = 1, and O(2)/S
1
= Z/2, SO(2) = S
1
.
(e) SU(n) = A U(n) : det A = 1 = SL
n
(C) U(n).
E.g., SU(2) =
__
z
1
z
2
z
2
z
1
_
: z
i
C, [z
1
[
2
+[z
2
[
2
= 1
_
spin group

= S
3
= z C
2
: [[z[[ = 1 C
2
= R
4
(homeomorphic).
SO(n) = A O(n) : det A = 1 = SL
n
(R) O(n).
E.g., SO(2)

= U(1), rotation of e
i
SO(3), rotations about various axes in R
3
.
SO(n), SU(n), U(n), O(n) are groups of isometries of geometric objects known as compact
Lie groups. Theory is done by H. Weyl, Classical Groups.
(15.3) Denition. A representation of a topological group on a nite-dimensional vector
space V is a continuous group homomorphism : G GL(V ) with the topology of GL(V )
inherited from the space EndV .
(There exist extensions when V is innite-dimensional see Telemann, remark 19.2.)
Here, continuous : G GL(V )

= GL
n
(C) means each g
_
(g)
_
ij
is continuous for i, j,
or the map GV V , (g, v) (g)v is continuous.
The compact group U(1)
(15.4) Theorem. The continuous homomorphisms C
1
GL
1
(C) = C

(i.e. the 1-dim.


representations of S
1
) are precisely the representations z z
n
(some n Z).
The proof is closely tied with Fourier Series. We need a couple of lemmas.
(15.5) Lemma. Consider (R, +). If : R R is a continuous homomorphism then
is multiplication by a scalar. (I.e., solve (x + y) = (x) + (y) for a continuous
function.)
Proof. Put c = (1). Then (n) = nc (n Z). Also m(1/m) = c, so (1/m) = c/m
(m Z). Hence (n/m) = cn/m. Thus (x) = cx (x Q), but Q is dense in R and
is continuous, so (x) = cx for all x R.
(15.6) Lemma. If : R
+
U(1) is a continuous homomorphism then there exists c R
with (x) = e
icx
for all x R.
46
Proof. Claim. There is a unique continuous homomorphism : R R such that (x) =
e
i(x)
(so we deduce (15.6) from (15.5)).
Recall that the exponential map : R
+
U(1), x e
ix
, maps the real line around
the unit circle with period 2.
,
0
,
2
R
+
E

d
d

`
_ ,
1
U(1)
complete the triangle!
For any continous : R
+
U(1) such that (0) = 1, there exists a unique continuous
lifting of this function to the real line such that (0) = 0 i.e., there exists a unique
continuous : R R such that (0) = 0 and (x) =

((x)) for all x.


(Lifting is constructed starting with condition (0) = 0 and then extending it a small
interval at a time. See Telemann, section 21. Non-examinable!)
Claim. If is a homomorphism then its lift is also a homomorphism. (So (x) = cx,
some c (15.4).)
We tensor (a +b) = (a)(b), hence
_
(a +b) (a) (b)
_
= 1. Hence (a +b)
(a) (b) = 2m for some m Z depending only on a, b. Varying a, b continuously,
m = constant; setting a = b = 0 shows m = 0.
Proof of (15.4). Given a representation : S
1
C

, it has a compact, hence bounded,


image. This image lies on the unit circle (integral powers of any other complex number
would form an unbounded sequence). Thus : S
1
S
1
is a continuous homomorphism.
Thus we get a homomorphism R S
1
, x (e
ix
), so by (15.6), there exists c R
with (e
ix
) = e
icx
.
Finally, 1 = (e
i2
) = e
i2c
, thus c Z. Putting n = c we have (z) = z
n
.
So
n
: U(1) C

, z z
n
, (n Z) give the complete list of irreducible representations of
U(1).
Schurs Lemma applies all irreducibles are 1-dimensional (cf. (4.4.)). Clearly their charac-
ters are linearly independent; in fact they are orthonormal under the inner product
, ) =
1
2
_
2
0
()() d ()
where z = e
i
. I.e., averaging over U(1). Finite linear combinations of these
n
are the
Fourier polynomials =

n
m=n
a
m

m
; the
n
are the Fourier modes.
U(1) is abelian, hence coincides with the space of conjugacy classes
(15.7) Theorem. (i) The functions
n
form a complete list of the irreducible representa-
tions of U(1).
(ii) Every nite-dimensional representation V of U(1) is isomorphic to a sum of the

n
. Its character

V
is a Fourier polynomial. The multiplicity of
n
in V equals

n
,

V
) (as in ()).
47
Remark. Complete reducibility of a nite-dimensional representation requires invoking
Weyls Unitary Trick (3.4

) to average over a given inner product using integration on U(1)


so before moving on to SU(2), lets consider a more general theory of compact groups.
General theory of compact groups
The main tools for studying representations of nite groups are:
Schurs Lemma holds here too
Maschkes Theorem. The relevant proof used Weyls trick of averaging over G. Need
to replace summation by integration over compact group G.
Namely, for each continuous function f on G, we have
_
G
f(g) dg C such that:

_
G
is a non-trivial linear functional

_
G
is left/right-invariant, i.e.
_
G
f(g) dg =
_
G
f(hg) dg =
_
G
f(gh) dg (h G)
G has total volume 1, i.e.
_
G
dg = 1
A (dicult) theorem of Haar asserts that these constraints determine existence and unique-
ness for any compact G. Well assume it, but for our Lie groups of interest (U(1), SU(2),
etc) there are easier proofs of existence.
(15.8) Examples. (a) G nite.
_
G
f(g) dg =
1
[G[

gG
f(g).
(b) G = S
1
.
_
G
f(g) dg =
1
2
_
2
0
f(e
i
) d.
(c) G = SU(2), 2 2 C-matrices preserving complex inner product and det = 1.
I.e, SU(2) =
__
u v
v u
_
: [u[
2
+[v[
2
= 1
_
.
Identify G with the unit 3-sphere S
3
C
2
= R
4
in such a way that left/right
translation by elements of G give isometries on the sphere. With this identication,
translation-invariant integration on G can be taken to be integration over S
3
with
usual Euclidean measure 1/2
2
(to normalise).
(d) Embed SU(2) H =
__
z
1
z
2
z
2
z
1
_
: z
i
C
_
, the quaternion algebra.
(Actually, its a division algebra, so that every non-zero element has an inverse.)
H is a 4-dimensional Euclidean space: [[A[[ =

det A = (x
2
1
+ x
2
2
+ x
2
3
+ x
2
4
)
1/2
,
where z
1
= x
1
+ix
2
, z
2
= x
3
+ix
4
, with SU(2) as the unit sphere in this normed
space.
Multiplication (from left or right) by an element of SU(2) is an isometry of H, viz:
(AX, AX) = det AX = det A det X = det X = (X, X) = (XA, XA),
where A SU(2).
48
Once we have found our translation-invariant integration on the set of continuous functions
on our compact group G, a lot can be proved about the representation theory of G in parallel
with nite groups.
Representations (continuous, nite-dimensional) characters (continuous functions G C).
Complete reducibility Weyls Unitary Trick of averaging over G replaced by integration.
Character inner product:

) =
_
G

(g)

(g) dg ()

irreducible i

) = 1.
Moreover,
(15.9) Theorem. For G compact.
(a) Every nite-dimensional representation is a direct sum of irreducible representa-
tions (so completely reducible).
(b) Schurs Lemma applies: if ,

are irreducible representations of G then


Hom(,

) =
_
C if is isomorphic to

0 otherwise
(c) The characters of irreducible representations form an orthonormal set with respect
to the inner product () above. (The set is innite, and it is not a basis for the
Hilbert space of all continuous class functions.)
Even showing completeness of characters is hard needs Peter-Weyl Theorem.
(d) If the characters of ,

are equal then



=

.
(e) If

is a character with

) = 1 then

is irreducible.
(f) If G is abelian then all irreducible representations are 1-dimensional.
Note. We dont have actions on nite sets: the regular representation is innite-dimensional.
Comment. The only spheres with continuous group homomorphisms are S
1
(= SO(2)) and
the 3-sphere S
3
(= SU(3)).
The group SU(2)
Recall G = SU(2) =
__
a b
b a
_
: a, b C, [a[
2
+[b[
2
= 1
_
.
G S
3
C
2
= R
4
,
_
a b
b a
_
(a
1
, a
2
, b
1
, b
2
). (Homeomorphism, i.e. continuous inverse.)
The centre is Z(G) = I.
Dene the maximal torus T =
__
a 0
0 a
_
: [a[
2
= 1
_
= S
1
.
49
Conjugacy
(15.10) Proposition. (a) Every conjugacy class ( of G = SU(2) meets T, i.e. ( T ,= .
(b) In fact, ( T =
_
x, x
1
if ( , = I
( if ( = I
(c) The normalised trace,
1
2
tr : SU(2) C, gives a bijection of the set of G-conjugacy
classes with the interval [1, 1], namely
g (
1
2
tr =
1
2
( +
1
) if g
_
0
0
1
_
Picture of ccls:
2-dim spheres of constant latitude
on unit sphere, plus the two poles

_
,
,
E
I
I
1
1
g
1
2
tr
Choose x
1
= c, then x
2
2
+x
2
3
+x
2
4
= 1 c
2
, so 1 < c < 1.
Given c (1, 1), all matrices g G have tr g = 2c.
Proof. Let S =
_
0 1
1 0
_
G, S
2
= I.
(a) Every unitary matrix has an orthonormal basis of eigenvectors, hence is conjugate
in U(2) to something in T, say QXQ
t
T.
We seek Q with det Q = 1 (so that Q SU(2)).
Let = det Q. Since QQ
t
= I, [[ = 1. If is a square root of then Q
1
= Q
SU(2) (since = 1/), hence Q
1
XQ
1
t
T.
(b) Let g SU(2) and suppose g (
G
. If g = I then ( T = g.
Otherwise g has distinct eigenvalues ,
1
and ( =
_
h
_
0
0
1
_
h
1
: h G
_
.
Thus ( T =
__
0
0
1
_
,
_

1
0
0
__
, by noting S
_

1
_
S =
_

_
.
Further, if
_

1
_
( then ,
1
= ,
1
, i.e. the eigenvalues are
preserved under conjugacy.
(c) Consider
1
2
tr : ccls [1, 1]. By (b) matrices are conjugate in G i their
eigenvalues agree up to order. Now
1
2
tr
_

1
_
=
1
2
( +
1
) = Re() = cos ( = e
i
)
hence the map is surjective onto [1, 1].
Its injective:
1
2
tr (g) =
1
2
tr (g

) then g, g

have the same characteristic polynomial,


viz X
2
tr (g)X + 1, hence the same eigenvalues, hence are conjugate.
Thus we write (
t
= g SU(2) :
1
2
tr (g) = t.
50
Representations
Let V
n
be the space of all homogeneous polynomials of degree n in the variables x, y.
I.e., V
n
= r
0
x
n
+ r
1
x
n1
y + . . . + r
n
y
n
, an (n + 1)-dimensional C-space, with basis
x
n
, x
n1
y, . . ., y
n
.
(15.11) GL
2
(C) acts on V
n
.
First, dene
n
: GL
2
(C) GL(V
n
)

= GL
n+1
(C). Let g =
_
a b
c d
_
.

n
(g)f(x, y) = f(ax +cy, bx +dy) = f
_
(x, y).g
_
(i.e., matrix product)
I.e., for f =

n
j=0
r
j
x
nj
y
j
, (g)f = r
0
(ax + cy)
n
+ r
1
(ax + cy)
n1
(bx + dy) + . . . +
r
n
(bx +dy)
n
.
Check that this denes a representation.
E.g. (a) n = 0,
0
= trivial
(b) n = 1, natural 2-dimensional representation.
1
_
a b
c d
_
has matrix
_
a b
c d
_
with
respect to the standard basis: x ax +cy, y bx +dy.
(c) n = 2,
2
=
_
a b
c d
_
has matrix
_
_
a
2
cb b
2
2ac ad +bc 2bd
c
2
cd d
2
_
_
with respect to the stan-
dard basis.
(We have (ax +cy)
2
+ (ax +cy)(bx +dy) + (bx +dy)
2
, so the rst column is the
coordinate vector of
2
(g)x
2
= (ax +cy)
2
= a
2
x
2
+ 2acxy +c
2
y
2
.)
Characters

Vn
(g) = tr
_

n
(g)
_
, g
_
z
z
1
_
T.

n
_
z
z
1
_
x
i
y
i
= (zx)
i
(z
1
y)
j
= z
ij
x
i
y
j
.
So
n
_
z
z
1
_
has matrix
_
_
_
_
_
z
n
z
n2
.
.
.
z
n
_
_
_
_
_
with respect to the standard basis.
Hence,

n
=

Vn
_
z
z
1
_
= z
n
+z
n2
+. . . +z
n
_
=
z
n+1
z
(n+1)
z z
1
unless z = 1.
_
(15.12) Theorem. The representations
n
: SU(2) GL(V
n
) of dimension n + 1 are
irreducible for n Z
0
.
Proof. Telemann (21.1) shows

n
,

n
) = 1 (implying

n
irreducible, by (15.9)(e)). We will
use combinatorics. Assume 0 ,= W V
n
, G-invariant.
Claim. If w =

j
r
j
x
nj
y
j
W with some r
j
,= 0, then x
nj
y
j
W.
51
Proof of claim. We argue by induction on the number of non-zero r
j
. If a unique
r
j
,= 0 then its clear (multiply by its inverse), so well assume more than one and
choose one.
Pick z C with z
n
, z
n2
, . . ., z
n
distinct in C.
Now,
n
_
z
z
_
w =

r
j
z
n2j
x
nj
y
j
W (G-space).
Dene w
i
=
n
_
z
z
1
_
w z
n2i
w W.
Then w
i
=

j
r

j
x
nj
y
j
and r

j
,= 0 (r
j
,= 0 and j ,= i). By induction
hypothesis, we have x
nj
y
j
W for all j with (r
j
,= 0 and j ,= i).
Finally, x
ni
y
i
= r
1
i
(w

r
j
x
nj
y
j
) W, so the claim is proved.
Now let 0 ,= w W. Wlog, w = x
nj
y
j
. It is now easy to nd matrices in SU(2), the
action of which will give all the x
ni
y
i
W.
E.g.,
1

2
_
1 1
1 1
_
: x
ni
y
i

2
(x +y)
ni
(x +y)
i
x
n
W
_
a b
b a
_
: x
n
(ax +by)
n

all x
ni
y
i
W
(a, b = 0) all coecients in here = 0
So all basis elements are in W. So W = V
n
.
Next we show that all irreducibles of SU(2) are of the form in (15.12).
Notation. Write N
0
[z, z
1
] =
_
n

m=n
a
m
z
m
: a
m
N
0
_
.
And N
0
[z, z
1
]
ev
= even Laurent polynomials, i.e. a
m
= a
m
for all odd m.
Let

=

V
be the character of some representation : G GL(V ). If g G = SU(2)
then g
G
_
z
z
1
_
for some z C. So

V
is determined by its restriction to T, hence

V
N
0
[z, z
1
] by ().
Actually

V
N
0
[z, z
1
]
ev
, as

V
_
z
z
1
_
=

V
_
z
1
z
_
, because
_
z
z
1
_

G
_
z
1
z
_
via S =
_
1
1
_
.
(15.13) Theorem. Every (nite-dimensional, continuous) irreducible representation of G
is one of the
n
: G GL(V
n
) above (n 0).
Proof. Assume : G GL(V ) is an irreducible representation aording the character

.
The characters characterise representations (15.9)(d), so its enough to show

=

n
for some n.
52
Now

0
= 1,

1
= z + z
1
,

2
= z
2
+ 1 + z
2
, . . . form a basis of Q[z, z
1
]
ev
, hence

a
n

n
, a nite sum with a
n
Q.
Clear the denominators and move all summands with negative coecients to the LHS:
m

iI
m
i

i
=

jJ
n
j

j
with I, J disjoint nite subsets of N, and m, m
i
, n
j
N.
The left and right hand sides are characters of representations of SU(2):
mV

I
m
i
V
i

=

J
n
j
V
j
.
Since V is irreducible we must have V

= V
n
, for some n J.
So we have found all irreducible representations of G; they are
n
: G GL(V
n
) (n ,= 0)
with V
n
the (n + 1)-dimensional space of homogeneous polynomials of degree n in x, y. The
characters of
n
are given by ().
To compute representations we just work with characters: as an example we derive a famous
rule for decomposing tensor products.
Tensor product of representations
Recall from section 9: if V, W are G-spaces we have V W aoridng

V W
=

V

W
.
Examples. V
1
V
1
= V
2
V
0
.
Character = (z +z
1
)
2
= z
2
+ 2 +z
2
= (z
2
+ 1 +z
2
. .
V2
) + 1
V0
V
1
V
2
= V
3
V
1
.
Character = (z +z
1
)(z
2
+ 1 +z
2
) = (z
3
+z +z
1
+z
3
) + (z +z
1
)
(15.14) Theorem (Clebsch-Gordan). V
n
V
m
= V
n+m
V
n+m2
. . . V
|nm|
Proof. Just check that the characters work.
Wlog n m and prove

n

m
=

n+m
+

n+m2
+. . . +

nm
.

n
(g)

m
(g) =
z
n+1
z
n1
z z
1
_
z
m
+z
m2
+. . . +z
m
_
=
m

j=0
z
n+m+12j
z
2jnm1
z z
1
=
m

j=0

n+m2j
(The n m ensures no cancellations in the sum.)
53
Some SU(2)-related groups
Check (see Telemann 22.1, and Examples Sheet 4 Question 6):
SO(3)

= SU(2)/I
SO(4)

= SU(2) SU(2)/(I, I)
U(2)

= U(1) SU(2)/(I, I)
()
(Isomorphisms, but actually homeomorphisms.)
So continuous representations of these groups are the same as continuous representations of
SU(2) and SU(2) SU(2), respectively, which send I and (I, I) to the identity matrix.
(15.15) Corollary. The irreducible representations of SO(3) are precisely
2m
: SO(3)
GL(V
2m
) (m 0).
Remarks. (a) We get precisely those V
n
with id in the kernel of the action, and id acts
on V
n
as
_
_
_
_
_
(1)
n
(1)
n2
.
.
.
(1)
n
_
_
_
_
_
= (1)
n
id
(b) V
2
is the standard 3-dimensional representation of SO(3). (The only 3-dimensional
representation in the list.)
(c*) For SO(4) the complete list is
m

n
(m, n 0, m n(2)) (see Telemann 22.7).
For U(2) the list is det
m

n
(m, n Z, n 0) where det : U(2) U(1) is
1-dimensional (see Telemann 22.9).
Sketch proof of () Recall from (15.8)(d) that SU(2) H

= R
4
can be viewed as the space
of unit norm quaternions. We also saw that multiplication from the left (and right) by
elements of SU(2) gives isometries of H. The left/right multiplication action of SU(2)
gives a homomorphism : SU(2) SU(2) SO(4), (g, h) : q gqh
1
.
Kernel. (g, h) sends 1 H to gh
1
, so (g, h) xes the identity i g = h, i.e. G =
(g, g) : g SU(2) = stab
SU(2)SU(2)
(1).
Now (g, g) xes every other quaternion i g Z(SU(2)), i.e. g = id. Thus ker =
(I, I).
Surjective and homeomorphic (i.e. inverse map is continuous). Restricting the
left/right action to G (the diagonal embedding of SU(2)) give the conjugation action
of SU(2) on the space of pure quaternions, i, j, k)
R
(the trace 0 skew-Hermitian 22
matrices). So get a 3-dimensional Euclidean space on which G acts, and (G) SO(3).
(G) = SO(3). Rotations in (i, j)-plane implemented by a + bk, similarly with any
permutations of i, j, k, and these rotations generate SO(3) (see some Geometry course).
So we have a surjective homomorphism SU(2) SO(3), and we know that ker =
id. The result follows.
Homeomorphism. Prove it directly or recall the fact that a continuous bijection
from a compact space to a Hausdor space is a homeomorphism (Sutherland 5.9.1)
54
Further worked example
S
n
, GL
2
(F
q
), H
p
.
We consider Heisenberg groups. For p prime, the abelian groups of order p
3
are C
p
3 , C
p
2 C
p
,
C
p
C
p
C
p
, and their character tables can be constructed using (4.5).
Suppose G is any non-abelian group of order p
3
. Let Z = Z(G), then its well-known that
Z ,= 1 and G/Z is non-cyclic, i.e. G/Z

= C
p
C
p
and Z = C
p
.
Take G = H
p
=
_
_
_
_
_
1
* *
0 1
*
0 0 1
_
_
:
*
F
p
_
_
_
, the modular Heisenberg group.
We take p odd (else G = D
8
or Q
8
).
Have Z = z), z =
_
_
1 0 1
0 1 0
0 0 1
_
_
.
With a =
_
_
1 1 0
0 1 0
0 0 1
_
_
and b =
_
_
1 0 0
0 1 1
0 0 1
_
_
, [a, b] = z and G

= Z.
There are p
2
linear characters (of degree 1) (recall G/G

= C
p
C
p
), and (p 1) characters
of degree p, induced from the 1-dimensional characters of the abelian subgroup
a, z) =
_
_
_
_
_
1
* *
0 1 0
0 0 1
_
_
_
_
_
of order p
2
.
Conjugacy classes
p conjugacy classes of size 1. The rest have size p and there are p
2
1 such classes.
Well show that the character table of H
p
looks like
p central ccls p
2
1 ccls each of size p
1 z . . . z
p1
a ab . . . a
1
b
1
p
2
linear
characters
1 1 . . . 1
char. table of
Cp Cp lifted
1 1 . . . 1
.
.
.
.
.
.
.
.
.
.
.
.
1 1 . . . 1
p 1 characters
of degree p
p p . . . . . .
p
p char.
table of Cp
0
.
.
.
p
55
More formally,
Z = z) gives p conjugacy classes of size 1: 1, z, . . ., z
p1
.
G/Z = aZ, bZ) = a
i
b
j
Z : 0 i p 1, 0 j p 1.
So, in particular, every element of G is of the form a
i
b
j
z
k
, 0 i, j, k, p 1.
the p
2
1 conjugacy classes of size p are ((a
i
b
j
) = a
i
b
j
z
k
: 0 k p1, (i, j) ,= (0, 0).
For aba
1
b
1
= z : aba
1
= zb (= bz as z central)
bab
1
= az
1
aa
i
b
j
a
1
= a
i
(aba
1
)
j
= a
i
b
j
z
j
ba
i
b
j
b
1
= (bab
1
)
i
b
j
= a
i
b
j
z
i
.
I.e., any conjugate of a
i
b
j
is some a
i
b
j
z
k
, as above.
Irreducible characters
(15.16) Theorem. As above, let G = a
i
b
j
z
k
: 0 i, j, k p 1 be a non-abelian group
of order p
3
. Write = e
2i/p

p
. Then the irreducible characters of G are:

u,v
(0 u, v p 1) (p
2
of degree 1)

u
(1 u p 1) (p
2
1 of degree p
where for all i, j, k,

u,v
(a
i
b
j
z
k
) = w
iu+jv

u
(a
i
b
j
z
k
) =
_
p
uk
if i = j = 0
0 otherwise
Proof. First, the p
2
linear characters. The irreducible characters of G
ab
= G/G

= G/Z =
C
p
C
p
are
u,v
(a
i
b
j
Z) =
iu+jv
(0 u, v p 1). The lift to G of
u,v
is precisely

u,v
.
Next, the p 1 characters of degree p.
Now, H =
_
_
_
_
_
1
* *
0 1 0
0 0 1
_
_
:
*
F
p
_
_
_

= a, z) is a normal abelian subgroup of index p.


Let
u
be a character of H dened as
u
(a
i
z
k
) =
uk
(0 k p 1), and calculate

G
u
. Choose transversal 1, b, . . ., b
p1
of H in G.

G
u
(a
i
z
k
) =
u
(a
i
) +
u
(a
i
z) +. . . +
u
(a
i
z
p1
)
=
u
(a
i
)
p1

r=0

u
(z
r
) (as homomorphic)
=
u
(a
i
)
p1

r=0

ur
= 0

G
u
(z
k
) =

u
(b
j
z
k
b
j
) = p
u
(z
k
) = p
uk
, and
u
(g) = 0 for all g / H.
Thus
G
u
=
u
.
56
Finally,

n
,
n
) =
1
p
3

gG

u
(g)
u
(g) =
1
p
3

gZ

u
(g)
u
(g)
=
1
p
3
p1

k=0

u
(z
k
)
u
(z
k
) =
1
p
3

z
p
2
= 1

Remarks. 1. Alternative is to apply Mackey (12.6).


2. Typically for p-groups: any irreducible representation is induced from a 1-dimen-
sional representation of some subgroup (Telemann, chapter 17).
3. For p odd, in fact there are two non-abelian groups of order p
3
:
G
1
= a, b : a
p
2
= b
p
= 1, b
1
ab = a
p+1
) with Z = a
p
)
G
2
= a, b, z : a
p
= b
p
= z
p
= 1, az = za, bz = zb, b
1
ab = az) with Z = z)
Notes typeset in L
A
T
E
X by Gareth Taylor
Please let me know of any corrections: glt1000@cam.ac.uk
57
PART II REPRESENTATION THEORY
SHEET 1
Unless otherwise stated, all groups here are nite, and all vector spaces are nite-dimensional
over a eld F of characteristic zero, usually C.
1 Let be a representation of the group G.
(a) Show that : g det (g) is a 1-dimensional representation of G.
(b) Prove that G/ ker is abelian.
(c) Assume that (g) = 1 for some g G. Show that G has a normal subgroup of
index 2.
2 Let : G F

be a 1-dimensional representation of the group G, and let : G


GL(V ) be another representation. Show that : G GL(V ) given by : g (g)(g)
is a representation of G, and that it is irreducible if and only if is irreducible.
3 Let G be the alternating group A
4
. Find all the degree one representations of G over F
for:
(a) F = C;
(b) F = R;
(c) F = Z/3Z.
[Hint: you can use the fact that the Klein 4-group V = {1, (1, 2)(3, 4), (1, 3)(2, 4), (1, 4)(2, 3)}
is the unique normal subgroup of A
4
(apart from the trivial subgroup and A
4
itself).]
Now let G = D
12
, the symmetry group of a regular hexagon. Let a G be a rotation
through /3 anticlockwise, and let b G be a reection, so that G = {a
i
, a
i
b : 0 i 5}.
Let A, B, C, D GL
2
(C) be the matrices
A =

e
i/3
0
0 e
i/3

, B =

0 1
1 0

, C =

1
2

3
2

3
2
1
2

, D =

1 0
0 1

.
Each of the following is a (matrix) representation of G over C (you need not verify this):

1
: a
r
b
s
A
r
B
s
.

2
: a
r
b
s
A
3r
(B)
s
.

3
: a
r
b
s
(A)
r
B
s
.

4
: a
r
b
s
C
r
D
s
.
Which of these are faithful? Which are equivalent to one another?
4 (Counterexamples to Maschkes Theorem)
(a) Let FG denote the regular FG-module (i.e. the permutation module coming from
the action of G on itself by left multiplication), and let F be the trivial module. Find all
the FG-homomorphisms from FG to F and vice versa. By considering a submodule of FG
isomorphic to F, prove that whenever the characteristic of F divides the order of G, there is
a counterexample to Maschkes Theorem.
(b) Find an example of a representation of some nite group over some eld of charac-
teristic p, which is not completely reducible. Find an example of such a representation in
characteristic 0 for an innite group.
1
2 PART II REPRESENTATION THEORY SHEET 1
5 Let N be a normal subgroup of the group G. Given a representation of the quotient
G/N, use it to obtain a representation of G. Which representations of G do you get this way?
Recall that the derived subgroup G

of G is the unique smallest normal subgroup of G


such that G/G

is abelian. Show that the 1-dimensional complex representations of G are


precisely those obtained from G/G

.
6 Let G be a cyclic group of order n. Decompose the regular representation of G explicitly
as a direct sum of 1-dimensional representations, by giving the matrix of change of coordinates
from the natural basis {e
g
}
gG
to a basis where the group action is diagonal.
7 Let G be the dihedral group D
10
of order 10,
D
10
= x, y : x
5
= 1 = y
2
, yxy
1
= x
1
.
Show that G has precisely two 1-dimensional representations. By considering the eect of y
on an eigenvector of x show that any complex irreducible representation of G of dimension
at least 2 is isomorphic to one of two representations of dimension 2. Show that all these
representations can be realised over R.
8 Let G be the quaternion group Q
8
of order 8,
Q
8
= x, y | x
4
= 1, y
2
= x
2
, yxy
1
= x
1
.
By considering the eect of y on an eigenvector of x show that any complex irreducible
representation of G of dimension at least 2 is isomorphic to the standard representation of
Q
8
of dimension 2.
Show that this 2-dimensional representation cannot be realised over R; that is, Q
8
is not
a subgroup of GL
2
(R).
9 State Schurs lemma.
Show that if G is a nite group with trivial centre and H is a subgroup of G with
non-trivial centre, then any faithful representation of G is reducible on restriction to H.
10 Let G be a subgroup of order 18 of the symmetric group S
6
given by
G = (123), (456), (23)(56).
Show that G has a normal subgroup of order 9 and four normal subgroups of order 3. By
considering quotients, show that G has two representations of degree 1 and four inequivalent
irreducible representations of degree 2. Deduce that G has no faithful irreducible representa-
tions.
PART II REPRESENTATION THEORY SHEET 1 3
11 In this question work over the eld F = R.
Let G be the cyclic group of order 3.
(a) Write the regular RG-module as a direct sum of irreducible submodules.
(b) Find all the RG-homomorphisms between the irreducible RG-modules.
(c) Show that the conclusion of Schurs Lemma (every homomorphism from an irre-
ducible module to itself is a scalar multiple of the identity) is false if you replace C by
R.
From now on let G be a cyclic group of order n. Show that:
(d) If n is even, the regular RG-module is a direct sum of two (non-isomorphic) 1-
dimensional irreducible submodules and (n2)/2 (non-isomorphic) 2-dimensional irreducible
submodules.
(e) If n is odd, the regular RG-module is a direct sum of one 1-dimensional irreducible
submodule and (n 1)/2 (non-isomorphic) 2-dimensional irreducible submodules.
[Hint: use the fact that RG CG and what you know about the regular CG-module
from question 6.]
12 Show that if is a homomorphism from the nite group G to GL
n
(R), then there is a
matrix P GL
n
(R) such that P(g)P
1
is an orthogonal matrix for each g G. (Recall that
the real matrix A is orthogonal if A
t
A = I.)
Determine all nite groups which have a faithful 2-dimensional representation over R.
SM, Lent Term 2011
Comments on and corrections to this sheet may be emailed to sm@dpmms.cam.ac.uk
PART II REPRESENTATION THEORY
SHEET 2
Unless otherwise stated, all groups here are nite, and all vector spaces are nite-dimensional
over a eld F of characteristic zero, usually C.
1 Let : G GL(V ) be a representation of G of dimension d, and aording character .
Show that ker = {g G | (g) = d}. Show further that |(g)| d for all g G, with
equality only if (g) = I, a scalar multiple of the identity, for some root of unity .
2 Let be the character of a representation V of G and let g be an element of G. If g is
an involution (i.e. g
2
= 1 = g), show that (g) is an integer and (g) (1) mod 2. If G is
simple (but not C
2
), show that in fact (g) (1) mod 4. (Hint: consider the determinant
of g acting on V .) If g has order 3 and is conjugate to g
1
, show that (g) (1) mod 3.
3 Construct the character table of the dihedral group D
8
and of the quaternion group Q
8
.
You should notice something interesting.
4 Construct the character table of the dihedral group D
10
.
Each irreducible representation of D
10
may be regarded as a representation of the cyclic
subgroup C
5
. Determine how each irreducible representation of D
10
decomposes into irre-
ducible representations of C
5
.
Repeat for D
12

= S
3
C
2
and the cyclic subgroup C
6
of D
12
.
5 Construct the character tables of A
4
, S
4
, S
5
, and A
5
.
The group S
n
acts by conjugation on the set of elements of A
n
. This induces an action
on the set of conjugacy classes and on the set of irreducible characters of A
n
. Describe the
actions in the cases where n = 4 and n = 5.
6 The group M
9
is a certain subgroup of the symmetric group S
9
generated by the two
elements (1, 4, 9, 8)(2, 5, 3, 6) and (1, 6, 5, 2)(3, 7, 9, 8). You are given the following facts about
M
9
:
there are six conjugacy classes:
C
1
contains the identity.
For 2 i 4, |C
i
| = 18 and C
i
contains g
i
, where g
2
= (2, 3, 8, 6)(4, 7, 5, 9), g
3
=
(2, 4, 8, 5)(3, 9, 6, 7) and g
4
= (2, 7, 8, 9)(3, 4, 6, 5).
|C
5
| = 9, and C
5
contains g
5
= (2, 8)(3, 6)(4, 5)(7, 9)
|C
6
| = 8, and C
6
contains g
6
= (1, 2, 8)(3, 9, 4)(5, 7, 6).
every element of M
9
is conjugate to its inverse.
Calculate the character table of M
9
. [Hint: You may nd it helpful to notice that
g
2
2
= g
2
3
= g
2
4
= g
5
.]
1
2 PART II REPRESENTATION THEORY SHEET 2
7 A certain group of order 720 has 11 conjugacy classes. Two representations of this group
are known and have corresponding characters and . The table below gives the sizes of the
conjugacy classes and the values which and take on them.
1 15 40 90 45 120 144 120 90 15 40
6 2 0 0 2 2 1 1 0 2 3
21 1 3 1 1 1 1 0 1 3 0
Prove that the group has an irreducible representation of degree 16 and write down the
corresponding character on the conjugacy classes.
8 The table below is a part of the character table of a certain nite group, with some
of the rows missing. The columns are labelled by the sizes of the conjugacy classes, and
= (1 + i

7)/2, = (1 + i

3)/2. Complete the character table. Describe the group in


terms of generators and relations.
1 3 3 7 7

1
1 1 1

2
3 0 0

3
3 0 0
9 Let x be an element of order n in a nite group G. Say, without detailed proof, why
(a) if is a character of G, then (x) is a sum of nth roots of unity;
(b) (x) is real for every character of G if and only if x is conjugate to x
1
;
(c) x and x
1
have the same number of conjugates in G.
Prove that the number of irreducible characters of G which take only real values (so-
called real characters) is equal to the number of self-inverse conjugacy classes (so-called real
classes).
A group of order 168 has 6 conjugacy classes. Three representations of this group are
known and have corresponding characters , and . The table below gives the sizes of the
conjugacy classes and the values , and take on them.
1 21 42 56 24 24
14 2 0 1 0 0
15 1 1 0 1 1
16 0 0 2 2 2
Construct the character table of the group.
[You may assume, if needed, the fact that

7 is not in the eld Q(), where is a primitive


7th root of unity.]
10 Let a nite group G act on itself by conjugation. Find the character of the corresponding
permutation representation.
11 Consider the character table Z of G as a matrix of complex numbers (as we did when
deriving the column orthogonality relations from the row orthogonality relations).
(a) Using the fact that the complex conjugate of an irreducible character is also an
irreducible character, show that the determinant det Z is det

Z, where

Z is the complex
conjugate of Z.
(b) Deduce that either det Z R or i. det Z R.
(c) Use the column orthogonality relations to calculate the product

Z
T
Z, where

Z
T
is
the transpose of the complex conjugate of Z.
(d) Calculate | det Z|.
PART II REPRESENTATION THEORY SHEET 2 3
12 The character table obtained in Question 9 is in fact the character table of the group
G =PSL
2
(7) of 22 matrices with determinant 1 over the eld F
7
(of seven elements) modulo
the two scalar matrices.
Deduce directly from the character table which you have obtained that G is simple.
[Comment: it is known that there are precisely ve non-abelian simple groups of order
less than 1000. The smallest of these is A
5

=PSL
2
(5), while G is the second smallest. It is
also known that for p 5, PSL
2
(p) is simple.]
Identify the columns corresponding to the elements x and y where x is an element of
order 7 (eg the unitriangular matrix with 1 above the diagonal) and y is an element of order
3 (eg the diagonal matrix with entries 4 and 2).
The group G acts as a permutation group of degree 8 on the set of Sylow 7-subgroups
(or the set of 1-dimensional subspaces of the vector space (F
7
)
2
). Obtain the permutation
character of this action and decompose it into irreducible characters.
Show that the group G is generated by an element of order 2 and an element of order 3
whose product has order 7.
[Hint: for the last part use the formula that the number of pairs of elements conjugate to x
and y respectively, whose product is conjugate to t, equals c

(x)(y)(t
1
)/(1), where
the sum runs over all the irreducible characters of G, and c = |G|
2
(|C
G
(x)||C
G
(y)||C
G
(t)|)
1
.]
SM, Lent Term 2011
Comments on and corrections to this sheet may be emailed to sm@dpmms.cam.ac.uk
PART II REPRESENTATION THEORY
SHEET 3
Unless otherwise stated, all groups here are nite, and all vector spaces are nite-dimensional
over a eld F of characteristic zero, usually C.
1 Recall the character table of S
4
from Sheet 2. Find all the characters of S
5
induced from
the irreducible characters of S
4
. Hence nd the complete character table of S
5
.
Repeat, replacing S
4
by the subgroup (12345), (2354) of order 20 in S
5
.
2 Recall the construction of the character table of the dihedral group D
10
of order 10 from
Sheet 2.
(a) Use induction from the subgroup D
10
of A
5
to A
5
to obtain the character table of
A
5
.
(b) Let G be the subgroup of SL
2
(F
5
) consisting of upper triangular matrices. Compute
the character table of G.
Hint: bear in mind that there is an isomorphism G/Z D
10
.
3 Let H be a subgroup of the group G. Show that for every irreducible representation
for G there is an irreducible representation

for H with a component of the induced


representation Ind
G
H

.
Prove that if A is an abelian subgroup of G then every irreducible representation of G
has dimension at most |G : A|.
4 Obtain the character table of the dihedral group D
2m
of order 2m, by using induction
from the cyclic subgroup C
m
. [Hint: consider the cases m odd and m even separately, as for
m even there are two conjugacy classes of reections, whereas for m odd there is only one.]
5 Prove the transitivity of induction: if H < K < G then
Ind
G
K
Ind
K
H

= Ind
G
H

for any representation of H.


6
(a) Let V = U W be a direct sum of CG-modules. Prove that both the symmetric
square and the exterior square of V have submodules isomorphic to U W.
(b) Calculate

and
S
2

, where is the irreducible representation of dimension 2 of


D
8
; repeat this for Q
8
. Which of these characters contains the trivial character in the two
cases?
7 Let : G GL(V ) be a representation of G of dimension d.
(a) Compute the dimension of S
n
V and
n
V for all n.
(b) Let g G and let
1
, . . . ,
d
be the eigenvalues of g on V . What are the eigenvalues
of g on S
n
V and
n
V ?
(c) Let f(x) = det(g xI) be the characteristic polynomial of g on V . Describe how to
obtain the trace

n
V
(g) from the coecients of f(x).
(d)* Find a relation between
S
n
V
(g) and the polynomial f(x). [Hint: rst do the case
when dimV = 1.]
1
2 PART II REPRESENTATION THEORY SHEET 3
8 Let G be the symmetric group S
n
acting naturally on the set X = {1, . . . , n}. For any
integer r
n
2
, write X
r
for the set of all r-element subsets of X, and let
r
be the permutation
character of the action of G on X
r
. Observe
r
(1) = |X
r
| =

n
r

. If 0 k n/2, show
that

k
,

= + 1.
Let m = n/2 if n is even, and m = (n 1)/2 if n is odd. Deduce that S
n
has distinct
irreducible characters
(n)
= 1
G
,
(n1,1)
,
(n2,2)
, . . . ,
(nm,m)
such that for all r m,

r
=
(n)
+
(n1,1)
+
(n2,2)
+ +
(nr,r)
.
In particular the class functions
r

r1
are irreducible characters of S
n
for 1 r n/2
and equal to
(nr,r)
.
9 If : G GL(V ) is an irreducible complex representation for G aording character ,
nd the characters of the representation spaces V V , Sym
2
(V ) and
2
(V ).
Dene the Frobenius-Schur indicator of by
=
1
|G|

xG
(x
2
)
and show that
=

0, if is not real-valued
1, if is real-valued.
[Remark. The sign +, resp. , indicates whether (G) preserves an orthogonal, respectively,
symplectic form on V , and whether or not the representation can be realised over the reals.
You can read about it in Isaacs or in James and Liebeck.]
10 If is a faithful character of the group G, which takes r distinct values on G, prove that
each irreducible character of G is a constituent of to power i for some i < r.
[Hint: assume that ,
i
= 0 for all i < r; use the fact that the Vandermonde r r matrix
involving the row of the distinct values a
1
, ..., a
r
of is nonsingular to obtain a contradiction.]
11 Construct the character table of the symmetric group S
6
. Identify which of your char-
acters are equal to the characters
(6)
,
(5,1)
,
(4,2)
,
(3,3)
constructed in question 8.
12 Show by induction on n that if the symmetric group S
n
with n 5 has a complex
irreducible representation of dimension d n then one of the following holds:
(i) d = 1, and is either the trivial representation 1 or the sign representation ;
(ii) d = n 1 and either 1 or 1 is the natural permutation representation;
(iii) n = 5 and d = 5(= n) or n = 6 and d = 5(= n 1).
[Hint: Restrict to S
n2
S
2
; it becomes reducible, unless it is linear; now use induction -
what do linear representations of S
n2
S
2
get induced to?]
SM, Lent Term 2011
Comments on and corrections to this sheet may be emailed to sm@dpmms.cam.ac.uk
PART II REPRESENTATION THEORY
SHEET 4
Unless otherwise stated, all vector spaces are nite-dimensional over C. In the rst nine
questions we let G = SU(2).
1 (a) Let V
n
be the vector space of complex homogeneous polynomials of degree n in the
variables x and y. Describe a representation
n
of G on V
n
and show that it is irreducible.
Describe the character
n
of
n
.
(b) Decompose V
4
V
3
into irreducible G-spaces (that is, nd a direct sum of irreducible
representations which is isomorphic to V
4
V
3
. In this and the following questions, you are
not being asked to nd such an isomorphism explicitly.)
(c) Decompose also V
2
3
,
2
V
3
and S
2
V
3
.
(d) Show that V
n
is isomorphic to its dual V

n
.
2 Decompose V
n
1
into irreducibles.
3 Determine the character of S
n
V
1
for n 1.
Decompose S
2
V
n
and
2
V
n
for n 1.
Decompose S
3
V
2
into irreducibles.
4 Let G act on the space M
3
(C) of 3 3 complex matrices, by
A : X A
1
XA
1
1
,
where A
1
is the 3 3 block diagonal matrix with block diagonal entries A, 1. Show that this
gives a representation of G and decompose it into irreducibles.
5 Let
n
be the character of the irreducible representation
n
of G on V
n
.
Show that
1
2
_
2
0
K(z)
n

m
d =
nm
,
where z = e
i
and K(z) =
1
2
(z z
1
)(z
1
z).
[ Note that all you need to know about integrating on the circle is orthogonality of characters:
1
2
_
2
0
z
n
d =
n,0
. This is really a question about Laurent polynomials. ]
6 (a) Let G be a compact group. Show that there is a continuous group homomorphism
: G O(n) if and only if G has an n-dimensional representation over R. Here O(n) denotes
the subgroup of GL
n
(R) preserving the standard (positive denite) symmetric bilinear form.
(b) Explicitly construct such a representation : SU(2) SO(3) by showing that SU(2) acts
on the vector space of matrices of the form
_
A =
_
a b
c a
_
M
2
(C) : A + A
t
= 0
_
by conjugation. Show that this subspace is isomorphic to R
3
, that (A, B) tr(AB) is a
positive denite non-degenerate invariant bilinear form, and that is surjective with kernel
{I}.
1
2 PART II REPRESENTATION THEORY SHEET 4
7 Check that the usual formula for integrating functions dened on S
3
R
4
denes an
G-invariant inner product on
G = SU(2) =
__
a b

b a
_
: a a + b

b = 1
_
,
and normalize it so that the integral over the group is one.
8 Suppose we are given that H is a subgroup of order 24 in G. We are told that H contains
{I} as a normal subgroup, and that the quotient group H/{I} is isomorphic to A
4
. Find
the character table of H. [You may assume that H has a conjugacy class containing six
elements of order 4, two conjugacy classes each containing four elements of order 3, and two
conjugacy classes each containing four elements of order 6.]
9 Compute the character of the representation S
n
V
2
of G for any n 0. Calculate
dim
C
(S
n
V
2
)
G
(by which we mean the subspace of S
n
V
2
where G acts trivially).
Deduce that the ring of complex polynomials in three variables x, y, z which are invariant
under the action of SO(3) is a polynomial ring. Find a generator for this polynomial ring.
10 The Heisenberg group of order p
3
is the (non-abelian) group
G =
_
_
_
_
_
1 a x
0 1 b
0 0 1
_
_
: a, b, x F
p
_
_
_
.
of 3 3 upper unitriangular matrices over the nite eld F
p
of p elements (p prime).
Show that G has p conjugacy classes of size 1, and p
2
1 conjugacy classes of size p.
Find p
2
characters of degree 1.
Let H be the subgroup of G comprising matrices with a = 0. Let : F
p
C

be a non-
trivial 1-dimensional representation of the cyclic group F
p
= Z/p, and dene a 1-dimensional
representation of H by

_
_
1 0 x
0 1 b
0 0 1
_
_
= (x).
Check that V

= Ind
G
H
is irreducible.
Now list all the irreducible representations of G, explaining why your list is complete.
11 Recall Sheet 3, q.8 where we used inner products to construct some irreducible characters

(nr,r)
for S
n
. Let n N, and let be the set of all ordered pairs (i, j) with i, j {1, 2, . . . , n}
and i = j. Let G = S
n
act on in the obvious manner (namely, (i, j) = (i, j) for S
n
).
Lets write
(n2,1,1)
for the permutation character of S
n
in this action.
Prove that

(n2,1,1)
= 1 + 2
(n1,1)
+
(n2,2)
+ ,
where is an irreducible character. Writing =
(n2,1,1)
, calculate the degree of
(n2,1,1)
.
Find its value on any transposition and on any 3-cycle. Returning to the character table of
S
6
calculated on Sheet 3 q.11, identify the character
(4,1,1)
.
SM, Lent Term 2011
Comments on and corrections to this sheet may be emailed to sm@dpmms.cam.ac.uk

Вам также может понравиться